Monday, March 30, 2009

JKMD 2009 Paper Questions with Explanations


(Click the Image to expand)




JKMD 2009
Thanks to Vivek sarmaji at Jammu
Students of 2008 batch who are the pride of DLI
MD Entrance TEST-PREP
TARGET BATCH 2009-10
DLI – Hyderabad
Jammu Address :17 A/C Gandhi Nagar
Ph No 2431418, 9419124645
Srinagar Address : Lane opp A Bee Book Store, Chanapora
Ph no. 9419014881

JKMD 2009
1. In the neural tube nerve cells are present in the :
A.Outer marginal layer
B.Middle mantle layer
C.Inner ependymal layer
D.In all the above layers
Ans b

As a result of cell division and migration, the neural tube becomes sub-divided into a ventricular (or ependymal) layer adjacent to the neural canal where mitotic activity continues, an intermediate (or mantle) layer composed mainly of migrating neuroblasts, and marginal zone under the outer limiting membrane in which newly born neurons are sending out neurites. As the neural tube matures, a longitudinal groove, the sulcus limitans extends the length of the spinal cord (to the diencephalon), separating dorsal from ventral halves. The dorsal portion, called the alar plate, receives input from sensory neurons; the ventral part, or basal plate, contains the cell bodies of many motor neurons.
The there Concentric layers are (1) an inner layer lining the central canal of the tube, (2) a middle cellular region, the mantle layer, and (3) an outer marginal layer consisting of fibers from the cells located in the first two layers.
Origin of the neural tube crest, illustrated by transverse from early human embryos.
As development continues, the cell bodies located in the mantle layer give a characteristic configuration to the tube. In transverse section, the cellular (mantle) region appears darker because of the closely packed nuclei and is now referred to as the gray matter of the tube. The marginal layer lacks cell bodies and contains fibers wrapped with a white, fatlike material, myelin. This layer becomes the white matter of the neural tube. The tube grows in diameter by the proliferation of cells from the inner layer and their migration into the mantle layer, and also by the firther deposition of myelin around the fibers.

2. In the suboccipital triangle to expose the third part of the vertebral artery you have to cut the:
A. Transverse ligament of the Atlas
B. Anterior atlanto-occipital membrane
C. Posterior atlanto-occipital membrane
D. Atlanto axial membrane
Ans c

• The suboccipital triangle is a region of the neck bounded by the following three muscles:
– Rectus capitis posterior major - above and medially
– Obliquus capitis superior - above and laterally
– Obliquus capitis inferior - below and laterally
• It is covered by a layer of dense fibro-fatty tissue, situated beneath the Semispinalis capitis.
• The floor is formed by the posterior occipito-atlantal membrane, and the posterior arch of the atlas.
• In the deep groove on the upper surface of the posterior arch of the atlas are the vertebral artery and the first cervical or suboccipital nerve
• The posterior atlantoöccipital membrane (posterior atlantoöccipital ligament), broad but thin, is connected above, to the posterior margin of the foramen magnum; below, to the upper border of the posterior arch of the atlas.
• On either side this membrane is defective below, over the groove for the vertebral artery, and forms with this groove an opening for the entrance of the artery and the exit of the suboccipital nerve.
• The free border of the membrane, arching over the artery and nerve, is sometimes ossified.
• The membrane is in relation, behind, with the Recti capitis posteriores minores and Obliqui capitis superiores; in front, with the dura mater of the vertebral canal, to which it is intimately adherent.
5. Examine the suboccipital triangle bounded by the rectus capitis posterior major medially, the obliquus capitis superior above and laterally, and the obliquus capitis inferior below. The floor of the suboccipital triangle is made up of the posterior atlantooccipital membrane above and the posterior arch of the atlas below.
Note:
that the greater occipital nerve (C2) winds round the lower border of the obliquus capitis inferior and that rectus capitis posterior minor lies medial to the rectus capitis posterior major.
Study the attachments of the suboccipital muscles:
The rectus capitis posterior major arises from the spine of the axis while the rectus capitis posterior minor takes origin from the posterior tubercle of the atlas. Both these muscles are inserted into the area below the inferior nuchal line, the major being lateral to the minor.
The obliquus capitis inferior arises from the spine of the axis and is inserted into the transverse process of the atlas.
The obliquus capitis superior arises from the transverse process of the atlas and is inserted into the area between the nuchal lines lateral to the semispinalis capitis.
Follow the third part of the vertebral artery medially under the posterior atlantooccipital membrane. Observe the plexus of veins in the suboccipital region.

3. Meniscofemoral ligaments are attached to the :
(A) Anterior horn of the lateral meniscus
(B) Anterior horn of the medial meniscus
(C) Posterior horn of the lateral meniscus
(D)Posterior horn of the medial meniscus
Ans c

The ligaments of Humphrey and Wrisberg are meniscofemoral ligaments which run from the posterior horn of the lateral meniscus to the lateral aspect of the medial femoral condyle;
- these ligaments are named based on their location in relation to the PCL;
- the anterior meniscofemoral ligament is known as the ligament of Humphrey where as the posterior meniscofemoral
ligament is known as the ligament of Wrisberg;
- in about 70 % of knees, there is either anterior meniscofemoral ligament of Humphrey or posterio meniscofemoral ligament of Wrisberg;
- latter is more common and is characterized by femoral origin merging w/ that of posterior cruciate ligament;
- in 6% of knees, both ligaments will be present;
- these meniscofemoral ligaments may play minor role as secondary restraints to posterior tibial translation after complete
transection of the posterior cruciate ligament;
Humphrey ligament: (anterior meniscofemoral ligament);
- is less than 1/3 the diameter of the PCL;
- arises from the posterior horn of the lateral meniscus, runs anterior to the to the PCL and inserts at th distal edge of the femoral PCL attachment;

- may be confused for the PCL during arthroscopy;
- in this situation, tug on the ligament while observing for motion of the lateral meniscus;
- Wrisberg’s ligament: (posterior meniscofemoral ligament);
- usually larger than ligament of Humphrey (upto 1/2 the diameter of the PCL diameter);
- extends from the posterior horn of lateral meniscus to medial femoral condyle;

4. The following facts are true in case of the vocaL cord, except :
(A) The mucous membrane of the vocal cord is firmly attached to the vocal ligament
(B) There is no submucous tissue over the vocal cord
(C) The vocal cord is pearly white in colour
(D)The vocal cord is lined by ciliated columner epithelium
Ans d

Vocal cords :
They are flat triangular bands and are pearly white in color
Oedema of the larynx cannot involve the true cords since there is no submucous tissue in which fluid can collect.
The lingual surface of the The lingual surface of the epiglottis and the true vocal cords are lned by stratified squamous epithelium, the remainder of the larynx is lined by respiratory epithelium; however, the layngeal surface of the epiglottis and false cords often develop patches of squamous metaplasia, especially in smokers and others likely to develop laryngeal lesions leading to biopsy. The ventricle and subglottis are virtually always lined by ciliated respiratory epithelium.

5. In the respiratory bronchiole, the lining epithelium contains the following cell, except :
(A) Goblet cell
(B) Clara cell
(C) Simple cuboidal cell
(D)Ciliated cuboidal cell
Ans d

• The respiratory bronchioles are the beginning of the respiratory segment of the airway and are just distal to the terminal bronchioles (which are the last segment of the conducting airway). The epithelium in this segment is simple cuboidal. The respiratory bronchioles are interrupted by alveoli which are thin walled evaginations. Alveolar ducts are distal continuations of the respiratory bronchioles.
• Clara cells are non-mucous and non-ciliated secretory cells found in the primary bronchioles of the lungs.
• Clara cells are dome-shaped and have short microvilli. One of the main functions of Clara cells is to protect the bronchiolar epithelium. They do this by secreting a small variety of products, including Clara cell secretory protein (CCSP) and a component of the lung surfactant. They are also responsible for detoxifying harmful substances inhaled into the lungs. Clara cells accomplish this with cytochrome P450 enzymes found in their smooth endoplasmic reticulum. Clara cells also multiply and differentiate into ciliated cells to regenerate the bronchiolar epithelium.
• The respiratory bronchioles represent the transition from the conducting portion to the respiratory portion of the respiratory system. The narrow channels are usually less than 2 mm in diameter and they are lined by a simple cuboidal epithelium, consisting of ciliated cells and non-ciliated Clara cells, which are unique to bronchioles. In addition to being structurally diverse, Clara cells are also functionally variable. One major function they carry out is the synthesis and secretion of the material lining the bronchiolar lumen. This material includes glycosaminoglycans, proteins such as lysozymes, and conjugation of the secretory portion of IgA antibodies. These play an important defensive role, and they also contribute to the degradation of the mucus produced by the upper airways. The heterogeneous nature of the dense granules within the Clara cell’s cytoplasm suggests that they may not all have a secretory function. Some of them may contain lysosomal enzymes, which carry out a digestive role, either in defense: Clara cells engulf airborne toxins and break them down via their cytochrome P-450 enzymes (particularly CYP4B1, which is only present in the clara cells) present in their smooth endoplasmic reticulum; or in the recycling of secretory products. Clara cells are mitotically active cells. They divide and differentiate to form both ciliated and non-ciliated epithelial cells.
• Clara cells contain Tryptase Clara, which is responsible for cleaving the hemagglutinin surface protein of influenza A virus, thereby activating it and causing the symptoms of flu.

6. The primitive streak appears in the caudal part of the germ disc at about :
(A)13th day of development
(B)15th day of development
(C)17th day of development
(D)18th day of development
Ans b

• The formation of the primitive streak is one of the first signs of gastrulation. It is characterized as a furrow in the midline of the embryonic disk at the future caudal end of the embryo. This furrow is formed by the ingression of epiblast cells which will go on to form the definitive endoderm and mesoderm by replacing hypoblast cells. The epiblast cells then develop into the definitive ectoderm.
• The primitive streak establishes a visible longitudinal axis of bilateral symmetry around which all embryonic structures will organize and align.
• A group of cancers, germ cell tumors, may be the result of an error during this stage of development.
Components
• The primitive streak consists of the following:
• Primitive groove - shallow valley extending along the length of the streak.
• Primitive knot - a raised circle of cells at the rostral tip of the primitive streak. (Hensen’s node)
• Primitive pit - a depression in the center of the primitive node
THE PRIMITIVE STREAK APPEARS AT THE BEFINNING OF THE THIRD WEEK
On about day 15 of development, a faint groove appears along the longitudinal midline of the germ disc, which has now assumed an oval shape Over the course of the next day, this goove vecomes deeper and elongrates to occupy about half the length of the embryo. On the day 16, a deeper depression surrounded by a slight mound of epiblast appears at the presumptive cranial end of the groove, nera the center of the germ disc. This groove, is called the primitive pit. And the mound surrounding it is called the primitive node. The entire structure is the primitive streak. The future head will from at the end of the germ disc nera the primitive pit, and the surface of the epiblast in the region adjacent to the midline will form the dorsal surface of the embryo. The appearance of the primitive streak establishes the lingitudinal axis and thus the bilateral symmerty of the future adult: the tisses to the right of this
Structure give rise to the right side of the body, and the tissues to the left of it give rise, in general, to the left side of the body. Although the form of the primary germ layers will change with folding of the embryo in the fourth week, the fundamental cranial/caudal. Left/right, and vertral/dorsal axes of the body are thus definitively established early in the third week of development.

7. Subsartorial plexus is formed by a twig from the nerves, except :
(A) Intermediate femoral cutaneous nerve
(B) Medial femoral cutaneous nerve
(C) Obturator nerve
(D)Saphenus nerve
Ans a

subsartorial plexus forms with branches of the saphenous and obturator nerves
• The medial cutaneous nerve (ramus cutaneus anterior; internal cutaneous nerve) passes obliquely across the upper part of the sheath of the femoral artery, and divides in front, or at the medial side of that vessel, into two branches, an anterior and a posterior. The anterior branch runs downward on the Sartorius, perforates the fascia lata at the lower third of the thigh, and divides into two branches: one supplies the integument as low down as the medial side of the knee; the other crosses to the lateral side of the patella, communicating in its course with the infrapatellar branch of the saphenous nerve. The posterior branch descends along the medial border of the Sartorius muscle to the knee, where it pierces the fascia lata, communicates with the saphenous nerve, and gives off several cutaneous branches. It then passes down to supply the integument of the medial side of the leg. Beneath the fascia lata, at the lower border of the Adductor longus, it joins to form a plexiform net-work (subsartorial plexus) with branches of the saphenous and obturator nerves. When the communicating branch from the obturator nerve is large and continued to the integument of the leg, the posterior branch of the medial cutaneous is small, and terminates in the plexus, occasionally giving off a few cutaneous filaments. The medial cutaneous nerve, before dividing, gives off a few filaments, which pierce the fascia lata, to supply the integument of the medial side of the thigh, accompanying the long saphenous vein. One of these filaments passes through the saphenous opening; a second becomes subcutaneous about the middle of the thigh; a third pierces the fascia at its lower third.
• The intermediate cutaneous nerve (ramus cutaneus anterior; middle cutaneous nerve) pierces the fascia lata (and generally the Sartorius) about 7.5 cm. below the inguinal ligament, and divides into two branches which descend in immediate proximity along the forepart of the thigh, to supply the skin as low as the front of the knee. Here they communicate with the medial cutaneous nerve and the infrapatellar branch of the saphenous, to form the patellar plexus. In the upper part of the thigh the lateral branch of the intermediate cutaneous communicates with the lumboinguinal branch of the genitofemoral nerve.

8. In mature graffian follicle the antrum folliculi lies in between :
(A) The theca externa and theca interna
(B) The theca interna and membrane granulose
(C) Within the membrane granulose
(D)Membrana granulose and zona pellucida
Ans d

• primordial follicle is a primary oocyte in the outer region of the cortex. It is
arrested in the first meiotic prophase. It is surrounded by flattened follicular
epithelial cells.
• The primary follicle is marked by the surrounding of flattened (squamous) follicular cells becoming cuboidal and the oocyte becomes bigger.
• The presence of the antrum is a characteristic of a secondary follicle. A follicle at this stage is also called an "antral follicle".
• The mature follicle is also called the Graffian follicle.

9. Geniculate ganglion of the facial nerve contains the neurons of :
(A) General somatic sensation
(B) Special somatic sensation
(C) General visceral sensation
(D)Special visceral sensation
Ans a,c,d

• GSA= general somatic afferent- pain, temp, touch, proprioception from body to CNS
• GSE= general somatic efferent- motor to skeletal muscles
• GVA= general visceral afferent- sensory from visceral to CNS
• GVE= general visceral efferent- motor to smooth muscle, cardiac muscle, and glands
• SSA= special somatic afferent- vision, hearing, and equilibrium to CNS
• SVA= special visceral afferent- smell and taste to CNS
• SVE= special visceral efferent- motor to muscles of head and neck
CNVII = Facial n. brain to face
– Both- SVE to muscles of facial expression
– GVE (parasympathetic) to lacrimal, submandibular, sublingual, nasal, and palatine glands
– SVA (taste) to anterior 2/3 of the tongue
– GVA from palate and nasal mucosa
– GSA from external acoustic meatus and auricle
• Parasympathetic cell body located in midbrain (superior salivatory nucleus), in nervus intermedius through internal acoustic meatus into geniculate ganglion, then two branches:
• 1) Greater Petrosal n. - GVE pre ganglionic parasympathetic, joins deep petrosal n. (sympathetic fibers) to form nerve of pterygoid canal going to pterygopalatine ganglion
• secretomotor to lacrimal, nasal, and palatine glands
• 2) From geniculate ganglion with motor nerves in facial canal, splits before exiting stylomastoid foramen into chorda tympani n., exits skull thru petrotympanic fissure and joins lingual n. in infratemporal fossa, to submandibular ganglion, synapse, post ganglionic go to submandibular, sublingual, and lingual glands
• Sensory (SVA) from anterior 2/3 of tongue travel with chorda tympani n. to join motor nerve roots in facial canal, to geniculate nucleus (where cell bodies are located), thru internal acoustic meatus, thru nervus intermedius into midbrain (solitary tract nucleus)

10. Sphenomandibular ligament is derived from the :
(A) Condensation of the investing layer of the deep cervical fascia
(B) Condensation of the prevertebral layer of the deep cervical fascia
(C) From the fibrous sheath of the mandibular arch
(D)From the fibrous sheath of the hyoid arch
Ans c

sphenomandibular ligament which, like the anterior malleolar, is derived from the perichondrial sheath of MeckeÃs cartilage
• The cartilaginous bar of the mandibular arch is formed by what are known as Meckel’s cartilages (right and left) ; above this the incus is developed.
• The dorsal end of each cartilage is connected with the ear-capsule and is ossified to form the malleus; the ventral ends meet each other in the region of the symphysis menti, and are usually regarded as undergoing ossification to form that portion of the mandible which contains the incisor teeth.
• The intervening part of the cartilage disappears; the portion immediately adjacent to the malleus is replaced by fibrous membrane, which constitutes the sphenomandibular ligament, while from the connective tissue covering the remainder of the cartilage the greater part of the mandible is ossified
• The sphenomandibular ligament (internal lateral ligament) is a flat, thin band which is attached above to the spina angularis of the sphenoid bone, and, becoming broader as it descends, is fixed to the lingula of the mandibular foramen. The function of the sphenomandibular ligament is to limit distension of the mandible in an inferior direction.
• Its lateral surface is in relation, above, with the lateral pterygoid; lower down, it is separated from the neck of the condyle by the internal maxillary vessels; still lower, the inferior alveolar vessels and nerve and a lobule of the parotid gland lie between it and the ramus of the mandible.
• Its medial surface is in relation with the medial pterygoid.
• The sphenomandibular ligament is most often damaged in an inferior alveolar nerve block.
• The ligament is derived from Meckel’s cartilage.
11. Spinocerebellar tract arises in the posterior horn of the spinal cord from the :
(A) Dorsomarginal nucleus
(B) Nucleus thoracis
(C) Nucleus proprius
(D)Substantia gelatinosa
Ans b

Spinocerebellar tracts. The dorsal spinocerebellar tract runs superficially in the lateral funiculus. It arises from cells in the ipsilateral Clarke’s Nucleus and runs from L3 on up. Follow this tract forward to a level just rostral to the pyramidal decussation and note that, at this level, it lies superficial to the spinal tract of V, the medullary and pontine equivalent of Lissauer’s tract. A little further rostral, at about mid-olivary levels, the dorsal spinocerebellar tract becomes incorporated into the inferior cerebellar peduncle. Although technically not a spinal cord tract, the cuneocerebellar tract should be mentioned here. It is basically the forelimb equivalent of the dorsal spinocerebellar tract. It arises from the accessory (external) cuneate nucleus, just lateral to the cuneate nucleus. Like the dorsal spinocerebellar tract, it is an uncrossed system.
• Clarke’s nucleus is a small section of grey matter located in lamina VII of the intermediate zone spinal cord which is found ventral to the gracile column and cuneate column and is involved in unconscious proprioception. It is found at the level of T1-L2 (lumbar vertebrae) on the spinal cord.
• Clarke’s nucleus is the origin of the dorsal spinocerebellar tract. Axons from cell bodies in the dorsal nucleus of Clarke travel through the ipsilateral lateral funiculus via the restiform body on their way to the cerebellum. Synonyms: nucleus thoracicus, Clarke’s column, Clarke’s nucleus

12. The following facts are true in case of G and Q bands of the chromosome, except :
(A) G band and Q band region are rich in A-T base pairs
(B) G band and Q band region are rich in G-C base pairs
(C) G band and Q band region correspond with the heterochromatin region
(D)In G band arid Q band regions DNA replication takes place slightly later
(E)
Ambiguous frame of MCQ

Ambiguous frame of MCQ
Banding Patterns
Chromosomes in metaphase can be identified using certain staining techniques, so called banding. Cellsare cultured and then stopped in metaphase to maximize the number of suitable cells. They are then spread on a slide, stained with a suitable dye and visualized in the microscope. Most conventional cytogenetic analyses depend on the karyotyping of banded metaphase chromosomes.
A band is defined as that part of a chromosome which is clearly distinguishable from its adjacent segments by appearing darker or brighter with one or more banding techniques. The chromosomes are visualized asconsisting of a continuous series of bright and dark bands.
The banding techniques fall into two principal groups: 1) those resulting in bands distributed along the length of the whole chromosome, such as G-, Q- and R-bandsand 2) those that stain a restricted number of specific bands or structures. These latter include methods whichreveal centromeric bands, C-bands, and nucleolusorganizer regions, NOR’s (at terminal regions of acrocentric chromosomes). C-banding methods donot permit identification of every chromosome in the somatic cell complement, but can be used to identify specific chromosomes.
• G- and R- bands can be bright field or fluorescent.
Bright field G-bands
• These G-bands are most commonly used. They take their name from the Giemsa dye, but can be produced with other dyes.
– In G-bands, the dark regions tend to be heterochromatic, late- replicating and AT rich.
– The bright regions tend to be euchromatic, early-replicating and GC rich.
Bright field R-bands
• These R-bands are approximately the reverse of G-bands (the R stands for "reverse"). The dark regions are euchromatic and the bright regions are heterochromatic.
Fluorescent G- and R-bands
• These bands are the photographic negative of the bright field versions. i.e. the reverse of the bright field G-bands and R-bands.
• Q-bands are like fluorescent G-bands, but certain heterochromatic regions are more brightly stained with Q-banding.
13. Chiasma formation takes place in the prophase of the 1st meiotic division :
(A)In leptotene stage
(B)In zygotene stage
(C)In pachytene stage
(D)In diplotene stage
Ans c

First Meiotic division
Prohase I is prolonged and shows the following stages.
Leptotene : The chromosomes are visible in the light microscope, as strings of irregular basophlic beads of DNA (chromomeres), separated by thread-like constrictions. Every chromosome has a specifically located centromere, the mechanical centre of the chromosome.
Zygotene : the chromosomes condense and come together in homologous pairs, spiralling on each other with a left hand sxrew, starting at the centromere the centromeres do not fuse.
Each chromosome in a pair is now replicated as two chromatids, spiralling with a right hand screw
Pachytene : further condensation and coiling of the chromosomes takes place, Chiasma formation is the reslut of crossing over en masse of chromosomal material
Diplotene : the paired chromosomes unravel, revealing in the microscope the presence of chiasmata, where crossing over of chromatids occurred
The centriole divides into two.
Diakinesis : further condensation reduces chromosomall length to one tenth or less of that in pachytene. Thw tow centromeres in any pair of chromosomes pull away from each other, producing "star" figures oriented in all directions The divided centriole outside the nuclear membrane sepaates into two polar centrioles. The first meiotic spindle forms between Them . The nuclear membrane is lost.
14. Inferior tibiofibular joint is : '
(A)Fibrous joint
(B)Cartilaginous joint
(C)Plane joint
(D)Pivot joint
Ans a

DISTAL TIBIOFIBULAR JOINT:
• It is a fibrous joint between lower ends of the tibia and fibula. There is no joint capsule. The interosseous ligament is strong and thick and binds the two bones together. The anterior and posterior ligaments are in front and behind the interosseous ligament.

• The inferior transverse ligament runs from the medial surface of the upper part of the lateral malleolus to the posterior border of the tibia. The nerve supply is from the deep peroneal and tibial nerves. Small amount of movement take place during movement of the anklejoint.


15. Glossopharyngeal nerve supplies :
(A)Palatopharyngeus muscle
(B)Salphingopharyngeus muscle
(C)Cricothyroid muscle
(D)Stylopharyngeus muscle
Ans d

There are a number of functions of the glossopharyngeal nerve:
• It receives sensory fibres from the posterior one-third ofthe tongue, the tonsils, the pharynx, the middle ear and the carotid sinus.
• It supplies parasympathetic fibres to the parotid gland via the otic ganglion.
• It supplies motor fibres to stylopharyngeus muscle, the only motor component of this cranial nerve.
• It contributes to the pharyngeal plexus.
• The palatopharyngeus or palatopharyngeal orpharyngopalatinus muscle is a long, fleshy fasciculus, narrower in the middle than at either end, forming, with the mucous membrane covering its surface, thepalatopharyngeal arch.
• The motor and most of the sensory supply of the pharynx is derived from the pharyngeal plexus of nerves on the surface of the pharynx.
• The plexus is formed by pharyngeal branches of the vagus (CN X) and glossopharyngeal (CN IX) nerves, and by sympathetic branches for the superior cervical ganglion.
• The motor fibres in the pharyngeal plexus are derived from the cranial root of accessory nerve (CN XI), and are carried by the vagus nerve to all muscles of the pharynx and soft palate.
• The exceptions are stylopharyngeus (supplied by CN IX) and the tensor veli palatini (supplied by CN V3).


16. Pterygopalatine fossa contains the following structures, except :
(A) Maxillary artery
(B) Maxillary nerve
(C) Greater superficial petrosal nerve
(D)Pterygopalatine ganglion
Ans c

• The pterygopalatine fossa is a fossa in the skull. It is the indented area medial to the pterygomaxillary fissure leading into the sphenopalatine foramen.

Contents
The pterygopalatine fossa contains
• the pterygopalatine ganglion suspended by nerve roots from the maxillary nerve
• the terminal third of the maxillary artery
• the maxillary nerve (CN V2), with which is the nerve of the pterygoid canal (a continuation of the facial nerve)
• The greater petrosal nerve is a nerve in the skull that branches from the facial nerve; it forms part of a chain of nerves that innervates the lacrimal gland. The fibres have synapses in the pterygopalatine ganglion.
• The greater petrosal nerve is a branch of the facial nerve that arises from the geniculate ganglion, a part of the facial nerve inside the facial canal. It enters the middle cranial fossa through the hiatus for the greater petrosal nerve (on the anterior surface of the petrous temporal bone). It proceeds towards the foramen lacerum, where it joins the deep petrosal nerve (sympathetic) to form the nerve of the pterygoid canal.
– The nerve of the pterygoid canal passes through the pterygoid canal to reach the pterygopalatine ganglion.


17. Chloride channels of neurons :
(A) are influenced by hormones
(B) are influenced by fluorines
(C) are decreased in alcoholism
(D)open during propagation of nerve impulse
Ans a
Alcohol increases chloride channel ion flow, relative to the post-synaptic chloride channel at rest, by engaging a GABAA receptor site
Sex hormones influence both the gene expression and functional activity of the chloride channel

18. Glutamine antagonists work as anticancer agents :
(A) by inhibiting purine nucleotide synthesis
(B) by blocking formation of AMP from IMP
(C) by inhibiting amidotransferase that transfers amide of glutamine to an acceptor molecule
(D)by inhibiting PRPP synthetase
Ans c
Acivicin is a glutamine antagonist that irreversibly inactivates many of the glutamine amidotransferases including formylglycinamidine ribonucleotide
• Glutamine amidotransferases (GATases) transfer the glutamine amide nitrogen to variety of substrates
– Most GATases can use ammonia (NH3) as an alternative nitrogen source. Sites for glutamine binding and for NH3­dependent synthesis are localised in different domains (sometimes in different subunits), termed ‘glutamine’ and ‘transferase’ domains.
– Phosphoribosyltransferases (PRTases) are involved in the biosynthesis and metabolism of nucleotides.
Two inhibitors of glutamine amidotransferases in the purine pathway, acivicin and triciribine have antineoplastic activity.
• glutamate N-methyl- -aspartate (NMDA) and á-amino-3-hydroxy-5-methyl-4-isoxazole-propionate (AMPA) antagonists inhibit the proliferation of human colon adenocarcinoma, astrocytoma, breast and lung carcinoma, and neuroblastoma cells in vitro.
• The antiproliferative effect of glutamate antagonists is Ca2+-dependent and results from decreased cell division and increased cell death. Glutamate antagonists produce morphological alterations in tumor cells, which consist of reduced membrane ruffling and pseudopodial protrusions, and decrease their motility and invasive growth

19. Gel filtration chromatography is a method for fractionation of proteins on the basis of :
(A)Size
(B)Charge
(C)Hydrophobicity
(D)Isoelectric point
Ans a
• Gel filtration chromatography seprarates proteins, prptides, and oligonucleotides on the basis of size

20. Phagocytosis involves :
(A) Production of Superoxide
(B) Production of Hydrogen Peroxide
(C) Production of Superoxide and Hydrogen Peroxide
(D)None of the above
Ans c
Upon phagocytosis, macrophagcs and neutrophils produce a variety of other toxic products that help kill the engulfed microorganism.The most important of these are the antimicrobial peptides and nitric oxide (NO), the superoxide anioti (O2"). and hydrogen peruxide (H2O2}. which are directly toxic to bacteria. Nitric oxide is produced by a high-output form of nitric oxide synthase, iNOS2. Superoxide is generated hy a, Multicomponent.

membrane-associated NAD I’M oxldase in a process known as the respiratory burst because it is accompanied by a transient increase in oxygen consump­tion; the superoxide is converted by the enzyme superoxide dismutase into H2O2 Further chemical and enzymatic reactions produce a range of toxic chemicals from H2O2. including the hydroxyl radical (*t)H) iind hypochlorite (OCI’J and hypobromile lOBr). Neutrophils are short-lived cells, dying soon after ihey have accomplished a round of phagocytosis. Dead and dying neulrophils are a major component of the1 pus that forms in some infections by extracellular bacteria, which are thus known as pus-forming or pyogenic bacteria. Macrophages. in contrast, are long-lived and continue to generate new lysosomes. I*aticnts with a disease called chronic granuloma-tous disease have a genetic deficiency’ of NADPI! oxidasc. which means that Iheir phagocytes do not produce the toxic oxygen derivatives characteristic of Ihe respiratory burst and so are less able to kill ingested microorganisms and clear an infection. People with this defect are unusually susceptible to bacte­rial and fungal infections, especially in infancy.

21. Which of the following does not increase as part of the acute phase response ?
(A)CRP
(B)Alfa 1 antitrypsin
(C)Albumin
(D)Fibrinogen
Ans c
• Acute-phase proteins are a class of proteins whose plasma concentrations increase (positive acute-phase proteins) or decrease (negative acute-phase proteins) in response to inflammation. This response is called the acute-phase reaction (also called acute-phase response).
• Inflammatory cells and red blood cellsIn response to injury, local inflammatory cells (neutrophil granulocytes and macrophages) secrete a number of cytokines into the bloodstream, most notable of which are the interleukins IL-1, IL-6 and IL-8, and TNF-á.
• The liver responds by producing a large number of acute-phase reactants. At the same time, the production of a number of other proteins is reduced; these are, therefore, referred to as "negative" acute-phase reactants
• Positive acute-phase proteins serve different physiological functions for the immune system.
– Some act to destroy or inhibit growth of microbes, e.g., C-reactive protein, Mannose-binding protein, complement factors, ferritin, ceruloplasmin, Serum amyloid A and haptoglobin.
– Others give negative feedback on the inflammatory response, e.g. serpins. Alpha 2-macroglobulin and coagulation factors affect coagulation.

22. Enzymes that catalyze the equilibrium between Aldoses and Ketoses are known as :
(A)Mutases
(B)Isomerases
(C)Aldolases
(D)Epimerases
Ans b
Isomerases have their own EC classification of enzymes: EC 5. Isomerases can be further classified into six subclasses:
• EC 5.1 includes enzymes that catalyze racemization (racemases) and epimerization (epimerases)
• EC 5.2 includes enzymes that catalyze the isomerization of geometric isomers (cis-trans isomerases)
• EC 5.3 includes intramolecular oxidoreductases
• EC 5.4 includes intramolecular transferases (mutases)
• EC 5.5 includes intramolecular lyases
• EC 5.99 includes other isomerases (including topoisomerases)
• An isomerase is an enzyme that catalyzes the structural rearrangement of isomers. Isomerases thus catalyze reactions of the form
where B is an isomer of A

23. The role of Citrate in fatty acid biosynthesis is :
(A) to activate fatty acid synthase
(B) to activate acetyl CoA carboxylase
(C) to act as a precursor for addition of carbon atom
(D)to inhibit oxidation of fatty acids
Ans b
Regulation of Acetyl-CoA Carboxylase by local metabolites:
• Palmitoyl-CoA, the product of Fatty Acid Synthase, promotes the inactive conformation of Acetyl-CoA Carboxylase (diagram above), diminishing production of malonyl-CoA, the precursor of fatty acid synthesis. This is an example of feedback inhibition.
• Citrate allosterically activates Acetyl-CoA Carboxylase. Citrate concentration is high when there is adequate acetyl-CoA entering Krebs Cycle. Excess acetyl-CoA is then converted via malonyl-CoA to fatty acids for storage.

24. Apolipoprotein B48 is :
(A) the main apoprotein of LDL
(B) the ^-terminal fragment of apo B 100
(C) the apoprotein of Lp (a)
(D)the oxidized form of apo B
Ans b
• ApoA-I is the main apolipoprotein associated with HDL
• Apolipoprotein B (APOB) is the primary apolipoprotein of low-density lipoproteins (LDL or "bad cholesterol"), which is responsible for carrying cholesterol to tissues
• The protein occurs in the plasma in 2 main isoforms, APOB48 and APOB100. The first is synthesized exclusively by the small intestine, the second by the liver.
– Both isoforms are coded by APOB and by a single mRNA transcript larger than 16 kb. APOB48 is generated when a stop codon (UAA) at residue 2153 is created by RNA editing. There appears to be a trans-acting tissue-specific splicing gene that determines which isoform is ultimately produced. Alternatively, there is some evidence that a cis-acting element several thousand bp upstream determines which isoform is produced.
• As a result of the RNA editing, APOB48 and APOB100 share a common N-terminal sequence, but APOB48 lacks APOB100’s C-terminal LDL receptor binding region. In fact, APOB48 is so called because it constitutes 48% of the sequence for APOB100.
• APOB 48 is a unique protein to chylomicrons from the small intestine. After most of the lipids in the chylomicron have been digested, APOB48 returns to the liver as part of the chylomicron remnant, where it is endocytosed and degraded.
• Two isoforms of apolipoprotein B (apoB)4 are coded for by the same gene by a novel mechanism in which apoB mRNA is edited by a cytidine to uridine conversion
• Editing results in changing the first base of the codon CAA, encoding glutamine 2153, to UAA, an in-frame translational stop codon. Consequently, apoB-48 is approximately half the size of apoB-100 and is identical to the amino-terminal half of apoB-100.

• Because the apoB-48 protein does not contain the carboxy-terminal half of the apoB-100, which is recognized by the LDL receptor, apoB-48 is not taken up by LDL receptor–mediated endocytosis. Instead, lipoproteins containing apoB-48 are cleared more quickly than those containing apoB-100 by a distinct receptor-mediated pathway involving apolipoprotein

25. Which of the following enzymes in the purine salvage pathway is deficient in severe Combined Immunodeficiency Syndrome ?
(A) Hypoxanthine-guanine phosphoribosyl transferase
(B) Xanthine oxidase
(C) Adenosine deaminase
(D)Aspartate transcarb amoylase
Ans b
X-linked severe combined immunodeficiency
• Most cases of SCID are due to mutations in the gene encoding the common gamma chain (ãc), a protein that is shared by the receptors for interleukins IL-2, IL-4, IL-7, IL-9, IL-15 and IL-21. These interleukins and their receptors are involved in the development and differentiation of T and B cells. Because the common gamma chain is shared by many interleukin receptors, mutations that result in a non-functional common gamma chain cause widespread defects in interleukin signalling. The result is a near complete failure of the immune system to develop and function, with low or absent T cells and NK cells and non-functional B cells.
• The common gamma chain is encoded by the gene IL-2 receptor gamma, or IL-2Rã, which is located on the X-chromosome. Therefore, immunodeficiency caused by mutations in IL-2Rã is known as X-linked severe combined immunodeficiency. The condition is inherited in an X-linked recessive pattern.
Adenosine deaminase deficiency
• The second most common form of SCID after X-SCID is caused by a defective enzyme, adenosine deaminase (ADA), necessary for the breakdown of purines. Lack of ADA causes accumulation of dATP. This metabolite will inhibit the activity of ribonucleotide diphosphate reductase, the enzyme that reduces ribonucleotides to generate deoxyribonucleotides. The effectiveness of the immune system depends upon lymphocyte proliferation and hence dNTP synthesis. Without functional ribonucleotide reductase, lymphocyte proliferation is inhibited and the immune system is compromised.

26. Which nucleotide in DNA forms dimers on exposure to ultraviolet light ?
(A)Adenine
(B)Guanine
(C)Cytosine
(D)Thymine
Ans d
• prevalent UV-derived DNA lesion, the cis,syn-thymine dimer (T x T),

27. Which form of DNA is most susceptible to oxidative damage in eukaryotic cells ?
(A)Nuclear DNA
(B)Mitochondrial DNA
(C)Telomeric DNA
(D)Satellite DNA
Ans c
The guanine rich repeats in telomeric DNA sequence make it highly susceptible to the formation of bulky structures (G-quadruplexes) and oxidative base

28. Ubiquitin is a protein required for :
(A)Protein degradation
(B)Glycoprotein degradation
(C)Arriino acid degradation
(D)Protein biosynthesis
Ans a
• Ubiquitin is a small, highly-conserved regulatory protein that is ubiquitously expressed in eukaryotes. Ubiquitination (or ubiquitylation) refers to the post-translational modification of a protein by the covalent attachment (via an isopeptide bond) of one or more ubiquitin monomers. The most prominent function of ubiquitin is labeling proteins for proteasomal degradation. Besides this function, ubiquitination also controls the stability, function, and intracellular localization of a wide variety of proteins. The ubiquitylation (or ubiquitination) cascade is started by the E1 enzyme.
The ubiquitination system functions in a wide variety of cellular processes, including[6]:
• Antigen processing
• Apoptosis

• Biogenesis of organelles
• Cell cycle and division
• DNA transcription and repair
• Differentiation and development
• Immune response and inflammation
• Neural and muscular degeneration
• Morphogenesis of neural networks
• Modulation of cell surface receptors, ion channels and the secretory pathway
• Response to stress and extracellular modulators
• Ribosome biogenesis
• Viral infection

29. Which one of the following will yield a net synthesis of glucose ?
(A)Acetoacetic acid
(B)Acetic acid
(C)Leucine
(D)Glutamic acid
Ans d
Glucogenic amino acids are:
• glycine
• serine
• valine
• histidine
• arginine
• cysteine
• proline
• alanine
• glutamate
• glutamine
• aspartate
• asparagine
• methionine

30. Which component of the Sarcomere has ATPase activity ?
(A)Myosin
(B)Actin
(C)Tropomyosin
(D)Troponin
Ans a
• Myosin is a protein having a molecular weight of ~ 470,000 daltons. There are about 300 molecules of myosin per thick filament. Each myosin contains two heads that are the site of the myosin ATPase, an enzyme that hydrolyzes ATP required for actin and myosin cross bridge formation.
– These heads interact with a binding site on actin

31. Which of the following sugars is commonly found at the non-reducing end of oli gosaccharides on plasma glycoproteins and mucins ?
(A)Glucose
(B)Sialic acid
(C)Mannose
(D)N-Acetyl glucosamine
Ans b
Sialic acid is,, often found at the nonreducing end of oligosaccharides

32. Which of the following cross-links are unique to elastins in extracellular matrix ?
(A)Allysine
(B)Lysinonorleucine
(C)Desmosine
(D)Glycyl glycine
Ans c
• A desmosine cross-link is formed from three allysyl side chains plus one unaltered lysyl side chain from the same or neighbouring polypeptides.
• It is found in elastin. Desmosine causes a yellow color.
• This desmosine is responsible for the rubber properties of elastin.

33. Death Sentence (Capital Punishment) can only be awarded by :
(A) First Class Judicial Magistrate
(B) Assistant Sessions Judge
(C) Additional Sessions Judge
(D)Chief Judicial Magistrate
Ans c
• Capital Punishment: The various methods of carrying out death sentence are: hanging, electro­cution, shooting, cyanide poisoning, lethal injection, garrotting, and guillotine.
What is 'Capita!punishment* ?
Capital punishment means death sentence.
Who can pass 'Death sentence* ?
Both Session Judge and Additional Session Judge.
What procedure is followed for execution of Death Sentence
It needs lo be confirmed by the Divi>;ii)n Bench of High Court, comprising of at least two Judges, How Death sentence is executed in India ? By hanging till death.

How Death sentence is executed in America ? by electrocution.
How Death sentence is executed in Germany^ Turkey ?

34. Membrana Pupillaris completely disappeared (atrophied) by :
(A) Just after birth
(B) 6 days after birth
(C) After 20 weeks of Intrauterine life
(D)After 28 weeks of Intrauterine life
Ans d
Between the sixth and seventh month, length of child 10 to 14 inches—that is, the length of the child after the fifth month is about double the lunar months—weight 1 to 3 pounds; skin, dusky red, covered with down their (lanugo) and sebaceous matter; member pupillaris disappearing; nails nor reaching to ends of fingers; meconium at upper part of large intestine; isles near kidneys; on appearance of convolutions in brain; points of ossification in four divisions of sternum.
At nine months, length of child 18 to 22 inches; weight, 7 to 8 pound?; skin rosy; lanugo only about shoulders; sebaceous matter on the body; hair on head about an inch long; rcstcs past inguinal ring; clitoris covered by the labia; membrane pupillaris disappeared; nails reach to ends of lingers; meconium At termination of large intestine; points of ossification in centre of cartilage at lower end of femur, about 1-1/2 to 2-1/2 lines in diameter; umbilicus midway between the ensiform cartilage and pubis.

35. Postmortem of a new born baby is done by opening first :
(A)The skull
(B)The chest cavity
(C)The abdominal cavity
(D)By modified T5 incision
Ans c

36. Most specific test to detect blood stains is :
(A)Spectroscopic test
(B)Benzidine test
(C)Absorption-elution test
(D)Teichman's test
Ans d
• Presumptive screening test (Is it blood?)
– Presumptive tests produce a color reaction or release of light
– Tests rely on catalytic properties of blood (hemoglobin presence)
Presumptive
5 Types of Color Tests
Benzidine
Phenolphthalein
O-Tolidine
Tetramethylbenzidine (TMB)
Leucomalachite Green (LMG)
Confirmation test (Seriously, is it blood?)
Confirmatory tests involve making crystals that detect the presence of hemoglobin
Species testing and DNA testing may also be considered confirmatory tests
Confirmatory Test
Teicbmario Test and Takayarna Test
Small amount of blood added to microscope slide
Chemical solution is added
Slide is heated to form crystals
Crystals viewed under microscope

37. Sexual perversion does not include :
(A)Indescent Assault
(B)Sodomy
(C)Transvestism
(D)Sadism
Ans b
SEXUAL PERVERSIONS
• Sexual perversions are persistently indulged sexual acts in which complete satisfacion is sought and obtained without sexual intercourse. The perverted behaviour is really a repulsively abnormal and frequently cruel form of tension release that by its nature tends to operate under compulsion. Most cases of sexual perversion involve early conditioning influences beyond the range of immediate knowledge. The mental state of the accused should be determined in all such cases by a psychiatrist.
• UNNATURAL OFFENCES
• Voluntary sexual intercourse against the order of nature with any man, or woman, or animal is an unnatural sex offence (S. 377, I.P.C.). These offences are punishable with imprisonment for life or up to ten years and also with fine.
(1) Natural offences
(II) Unnatural offences:
(I) Sodomy.
(2) Tribadism.
(3) Bestiality, and
(4) Buccal coitus.
(III) Sexual perversions:
(1) Sadism.
(2) Masochism.
(3) Necrophilia.
(4) Fetichism.
(5) Transvestism.
(6) Exhibitionism.
(7) Masturbation.
(8) Voyeurism.
(9) Frotteurism.
(10) Undinism, etc.

38. Which of the following poisoning resembles Thyrotoxicosis ?
(A)Thalium
(B)Bi-nitro compounds
(C)Bromide
(D)Chloral hydrate
Ansb
Dinitro compounds poisoning resembles thyrotoxicosis

39. Acrodynia is seen in poisoning by :
(A)Lead
(B)Zinc
(C)Mercury
(D)Arsenic
Ans c
• Acrodynia (mercury poisoning).

40. Ultra-Violet Fluorescence of limb bones last after :
(A)200 years
(B)300 years
(C)100 years
(D)400 years
Ans c
1) A nitrogen content in excess of 3.5 g per cent indicates that the bone age is less than SO years ('modern*) and a nitrogen content of 2.5 g per cent or less suggests that the: 1M me is more than 350 years old,
2) Seven or more arninoacids can l>e demon started if the bone is less than 100 years did; proline and hydroprolinc are invariably pa-sent upto 50 years.
3) The entire cut surface nf freshly sawn hone will show ultraviolet fluorescence in case of 'modern bone'. As the age advances the fluore scence starts disappearing from the periphery. How ever traces mav be detected as late as 1800 years. H
4) Powdered bone gives a positive benzidine test upto 100-150 years, However since false positive tests arc common, only the negative test is of value.
5) Immunologies! activity with anti-human serum as demonstrated by the gel-diffusion Techni que ceases in five years' time. Berg (1963) states that the activity is continued upto 20 years and a dcla)xxl weak reaction can be seen even upto 50 years.
Bones features in the time period over 40-50 to 100 years
• Such kind of bones usually having survived since the War#2 or the post-war period are submitted to the laboratory very often. The main features of them can be described as follows:
• - the bones are light in weight, dry, the color varies from dark yellow to brownish or brown;
• - the surface contains many scarifications and infractions, the stratification of the skull vault exterior plate is observed;
• - the porous tissue and marrow cavities are empty or contain remnants of a blackish crumbling mass,
• - in the cross-section the compact tissue in the depth of 0,5-1,5 mm is mineralized, whereas in deeper layers the bone structure is well preserved,

• - microscopically the blood vessels contain no blood and Haversian channels look empty,
• - in the ultraviolet light the compact tissue always fluorescence well with a bluish-violet, bright violet or greyish - violet color.

41. Fingerprints can be determined upto depth of :
(A)0.4 mm
(B)0.5 mm
(C)0.6 mm
(D)0.8 mm

42. The following also known as "Micky Finn" :
(A)Ethylene Glycol
(B)Methyl Alcohol
(C)Potassium Bromide
(D)Chloral Hydrate
Ans d
• mixture of chloral hydrate and alcohol is thought to have been the original Mickey Finn

43. Common cause of Neonatal meningitis is
(A)Streptococcus MG
(B)Streptococcus faecalis
(C)Streptococcus agalactiae
(D)Streptococcus equisimilis
Ans c
• GBS is the most common cause of neonatal meningitis in the United States, followed by E coli, Listeria monocytogenes, and Klebsiella pneumoniae.
• In developing countries, Staphylococcus aureus and Streptococcus pyogenes, along with GBS, are the most common gram-positive bacteria causing neonatal meningitis; Salmonella species and E coli are the most common gram-negative bacteria causing neonatal meningitis.GBS=groupB streptococci
Group B Streptococcus, also known as Streptococcus agalactiae

44. The spore of Clostridium tetani is :
(A)Terminal and elliptical
(B)Subterminal and elliptical
(C)Subterminal and round
(D)Terminal and round
Ans d
• Most Clostridium spp., including C. perfringens and C. botulinum, have ovoid subterminal. (OST) spores.
– C. tetani have round terminal (RT) spores
• C. tetani is a rod-shaped, obligate anaerobe which stains Gram positive in fresh cultures; established cultures may stain Gram negative.[1] During vegetative growth, the organism cannot survive in the presence of oxygen, is sensitive to heat and has flagella which provide limited mobility. As the bacterium matures, it develops a terminal spore, which gives the organism its characteristic appearance. C. tetani spores are extremely hardy, and are resistant to heat and most antiseptics

45. Sereny Test is carried out for identification of :
(A)ETEC
(B)EHEC
(C)EIEC
(D)EPEC
Ans c
• Invasive E coli, like Shigella, causes a rapid keratoconjunctivitis when placed on the conjunctiva of the guinea pig eye (Sereny test

46. The example of immediate Hypersensitivity is :
(A)Casoni test
(B)Lepromin test
(C)Tuberculin test
(D)Frei test
Ans a
• Frei’s test was developed in 1925 for lymphogranuloma venereum (LGV) and uses Frei’s antigen. It indicates delayed hypersensitivity to an intradermal standardized antigen prepared from chlamydia grown in the yolk sac of a chick embryo.
– The test is read after 48h and again on the fourth day. A nodule more than 5mm at fourth day is considered a positive response.
A positive test indicates past or present chlamydial infection. Frei’s test becomes positive two to eight weeks after infection. Frei’s antigen is common to all chlamydial species and is not specific to LGV. Due to its nonspecific nature, the test is no longer used.
• Casoni test detects an in vivo hypersensitivity which can be both of immediate and delayed type

47. J-Chain is present in :
(A)IgG
(B)IgA
(C)IgD
(D)IgE
Ans b
• J chain is a protein component of the antibodies IgM and IgA.
• Because IgM and IgA are the only two types of antibody that polymerize, initial hypotheses stated that J chain was required for polymerization. However, it was subsequently found that IgM is able to polymerize in the absence of J chain as both a pentamer and a hexamer.
• Current hypotheses indicate that J chain may be necessary to enable secretion of antibodies into mucosal tissues.

48. Staphylococcus is a :
(A) Gram positive cocci in chain
(B) Gram positive cocci in cluster
(C) Gram negative cocci in chain
(D)Gram negative cocci in cluster
Ans b
Important phenotypic characteristics of Staphylococcus aureus.
Gram-positive, cluster-forming coccus nonmotile, nonsporeforming facultative anaerobe

49. Bird flu is caused by an Avian subtype Influenza structure :
(A)H1N5
(B)H5N1
(C)H1N1
(D)H3N2
Ans b
• The H5N1 virus can travel from farm to farm in the mud of a farmer’s truck or in the dust on his shoes. It can survive on the bars of cages that may be used in the commercial transport of live animals
• when outbreaks involve highly pathogenic forms of H5 and H7 viruses, mortality is close to 100 percent among infected poultry, leading to massive culling in efforts to keep the virus from spreading.
• There are 15 different HA subtypes and nine different NA subtypes. Many different combinations of HA and NA proteins are possible.
• Only some influenza A subtypes ( H1N1, H1N2, and H3N2) are currently in general circulation among people. Other subtypes are found most commonly in other animal species, for example,
• H7N7 and H3N8 viruses affect horses.

50. Which one of the bacteria is transmitted by sexual route ?
(A)Haemophilus aegyptius
(B)Haemophilus ducreyi
(C)Haemophilus haemolyticus
(D)Haemophilus parahaemolyticus
Ans b
Haemophilus ducreyi is a fastidious Gram-negative coccobacillus causing the sexually transmitted disease chancroid

51. Cytotoxic (Type II) hypersensitivity is mediated by ;
A.IgG
B.IgA
C.IgD
D.IgE
Ans a
Type II hypersensitivity. Antigens on the patient’s own cell surfaces. Ig G & Ig M mediated. complement activation

52. Widal test is a/an :
(A)Agglutination test
(B)CFT
(C)Precipitation test
(D)Flocculation test
Ans a
• The Widal test is a presumptive serological test for Enteric fever or Undulant fever.
• In case of Salmonella infections, it is a demonstration of agglutinating antibodies against antigens O-somatic and H-flagellar in the blood.
• For brucellosis, only O-somatic antigen is used. It’s not a very accurate method, since patients are often exposed to other bacteria (e.g. Salmonella enteritidis, Salmonella typhimurium) in this species that induce cross-reactivity; many people have antibodies against these enteric pathogens, which also react with the antigens in the Widal test, causing a false-positive result. Test results need to be interpreted carefully in the light of past history of enteric fever, typhoid vaccination, general level of antibodies in the populations in endemic areas of the world. Typhidot is the other test used to ascertain the diagnosis of typhoid fever. As with all serological tests, the rise in antibody levels needed to make the diagnosis takes 7-14 days, which limits their use. Other means of diagnosing Salmonella typhi (and paratyphi) include cultures of blood, urine and faeces. The organism also produces H2S from thiosulfate.

53. Cysticerosis is caused by ingestion of :
(A)Cysticercus cellulosae
(B)Cysticercus bovis
(C)Egg of Taenia saginata
(D)Egg of Taenia solium
Ans d
• Cysticercosis is caused by the metacestode, or larval, stage of Taenia solium, the pork tapeworm

• When humans ingest undercooked pork that contains cysticerci of T solium, the scolex evaginates from the cyst and develops into an intestinal tapeworm. The tapeworm grows to a length of up to 10 meters and has hundreds of proglottids. Mature proglottids contain approximately 50,000 eggs each. Free eggs or whole proglottids are released periodically into the stool of the carrier and can survive in the environment for many months.
• When pigs ingest the proglottids or eggs, the eggs hatch, penetrate the pigs’ intestinal wall, and spread to skeletal muscle, especially the neck, tongue, and trunk. There, the larvae mature into encysted cysticerci over 2-3 months. The cysticerci suppress the host inflammatory response and survive in tissues for months to years. The life cycle is completed when humans ingest inadequately cooked pork that contains viable cysticerci or ingest eggs. Humans are end hosts of the larval stage and develop cysticercosis similar to that in pigs. Ingestion of encysted pork does not directly cause cysticercosis; rather, it produces an intestinal infection of the adult tapeworm and a carrier state for the T solium eggs that, when ingested by humans, produce the clinical syndrome of cysticercosis.
Humans can be infected with eggs through fecal-oral transmission or possibly through autoinfection. Fecal-oral contamination usually occurs via infected food handlers who do not appropriately wash their hands before working or via ingestion of fruit and vegetables fertilized with contaminated human waste. The eggs are sticky and can often be found under the fingers of tapeworm carriers. Thus, even populations who do not eat pork can develop cysticercosis. The egg-containing feces can contaminate water supplies in endemic areas. If the water is used to irrigate fruits and vegetables, eggs are ingested with the contaminated food. Thus, people who have never visited endemic countries can also develop infection. Autoinfection involves the retrograde transmission of proglottids from the intestines into the stomach with subsequent release of T solium eggs into the gut.
Human neurocysticercosis can result after ingestion of food contaminated with T solium eggs. The cystic larval stage that normally occurs in pigs develops in the human host and spreads to the skeletal muscle and brain. In this situation, the human becomes the end intermediate host.

54. infective form of malaria is :
(A)Sporozoite
(B)Merozoite
(C)Schizont
(D)Gametocyte
Ans a

55. Which form will not be present commonly in peripheral blood smear in Malignant Tertian Malaria ?
(A)Trophozoite
(B)Schizont
(C)Male gametocyte
(D)Female gametocyte
Ans b
• The appearance of the parasite varies in the thick and thin films. The thick unfixed film shows only leucocytes and parasites; erythrocytes are destroyed in the staining process. The parasites themselves are also altered. Young trophozoites appear as incomplete rings or spots of blue cytoplasm with discrete red nuclei. In mature trophozoites, the cytoplasm may be fragmented, and the various characteristics of the different species are often indistinct. Gametocytes and schizonts usually retain their characteristic appearances. Thin film examination is essential for the accurate identification of plasmodial species, which has an important bearing on treatment.
Cerebral malaria: late trophozoites show a coarse granule of pigment in peripheral blood
• The microscopic diagnosis of P. vivax is based on the following:
– a) the infected red cells are enlarged and deformed;
– b) the schizont shown contains 20 merozoites (schizonts of P. malariae and P. ovale have fewer merozoites; and in P. falciparum schizonts are not usually seen in the peripheral blood);
– c) the round gametocyte shown, contained in an enlarged red cell.
– (In this case, the typical Schüffner’s dots were not visible, probably due to staining problems.)

56. Granulomatous amoebic encephalitis is caused by
(A)Balamuthia mandrillaris
(B)Naegleria fowleri
(C)Entamoeba histolytica
(D)Entamoeba coli
• Granulomatous amoebic encephalitis is a central nervous system disease caused by certain species of amoeba, especially Balamuthia mandrillaris.
• GAE may present in numerous ways. There is no solid definition, as only a handful of patients have presented thus far with GAE. GAE can present with: focal paralysis, seizures, brainstem symptoms, and other neurological problems, some of which may mimic glioma (especially brainstem glioma), or other brain diseases, which may hamper timely diagnosis. These symptoms are caused by inflammatory necrosis of brain tissue brought on by amoebic infiltrates.

57. Occult filariasis is caused by :
(A)Adult Wuchereria bancrofti
(B)Microfilaria bancrofti
(C)Adult Brugia malayi
(D)Microfilaria malayi
Ans b
• The term Occult Filariasis is commonly used to designate filarial infections in which mf are not found in the periphral blood although they may be seen in tissues. However, it has now been shown that in some cases with occult filariasis, mf may actually be found after more careful blood examination despite their low density. Occult filariasis is believed to result from a hypersensitivity reaction to filarial antigens derived from microfilariae.Only a very small proportion of individuals in a community where filariasis is endemic develop occult forms of the disease.
The clinical manifestations of Occult filariasis are:
• Tropical Pulmonary Eosinophilia (TPE)
• Glomerulopathies(Granulonephritis)
• Endomyocardial fibrosis
• Filarial Arthritis
• Filarial granulomas in the breast
• It is found that 90% of the patients with filarial arthritis tested are positive for filarial antibodies when tested with mf of W.bancrofti in a fluorescent antibody test

58. The segment of RNA genome in influenza virus type C is
(A)Six
(B)Seven
(C)Eight
(D)Nine
Ans c
• Virions of the genus Influenzavirus A, Influenzavirus B and Influenzavirus C virions contain single stranded, negative sense, segmented (7-8), RNA (ssNSRNA) genome.

59. Negri bodies are :
A.Intracytoplasmic eosinophilic
B.Intracytoplasmic basophilic
C.Intranuclear eosinophilic
D.Intranuclear basophilic
Ans a
Negri body is pathognomonic.
They are oval eosinophilic neuronal cytoplasmic inclusion measuring 1-7 micron in diameter

60. True yeast is :
(A)Candida albicans
(B)Pityrosporum orbiculare
(C)Cryptococcus neoformans
(D)Cladosporium wernickli
Ans a

61. Concentration of H2SO4 in Z-N staining is :
(A)1%
(B)5%
(C)10%
(D)20%
Ans d
Procedure
• 1. Cover with tissue paper
• 2. Flood slide with carbolfuchsin, the primary stain, for 2 minutes while heating with steam.

• 3. Remove paper cover, decolorize slide with a mixture of hydrochloric acid and ethanol.
• 4. Counter stain with methylene blue.
Modifications
• 5% Sulphuric Acid is used for staining Mycobacterium leprae instead of the 20% used for Mycobacterium tuberculosis
• Kinyoun modification (or cold Ziehl-Neelsen technique) is also available.

62. Which one is not dermaophytes ?
(A)Trichophyton
(B)Microsporum
(C)Epdermophyton
(D)Malssezia furfur
Ans d
• A dermatophyte is a parasitic fungus (mycosis) that infects the skin.
• The term embraces the imperfect fungi of the genera Epidermophyton, Microsporum and Trichophyton.

63.VMMR is a :
(A) Double dose vaccine given by S.C. route
(B) Single dose vaccine given by S.C. route
(C) Double dose vaccine given by I.M. route
(D)Single dose vaccine given by I.M. route
Ans a
MMR are recommended by the manufacturers to be administered via the subcutaneous route
• Why is a second dose of MMR necessary?
– About 2%-5% of persons do not develop measles immunity after the first dose of vaccine. This occurs for a variety of reasons. The second dose is to provide another chance to develop measles immunity for persons who did not respond to the first dose.

64. Which is not associated with Rheumatic heart disease ?
(A)Jones criteria
(B)Dukes criteria
(C)Anitschkow cell
(D)Pancarditis
Ans b
Dukes Criteria for Infective endocarditis
Major
• Typical organism in 2 separate cultures or persistently +ve blood cultures (o, D‘ hrs apart)
• +ve echocardiogram (vegetation, abscess) or new valvular regurgitation
Minor
• Predisposition
• Fever ^‘°C
• Vascular/ immunological phenomena (splinter haemorrhages, Osler’s nodes)
• +ve blood cultures (not meet major criteria)
• +ve echocardiogram (not meet major criteria)

65. All are HIV – Associated neoplasm, except :
(A)Kaposi’s sarcoma
(B)B-Cell non-Hodgkin lymphoma
(C)Primary lymphoma of brain
(D)Nasopharyngeal carcinoma
Ans d
HIV-associated CNS lymphoma is a diffuse, large-cell non-Hodgkin lymphoma that usually occurs in the brain

66. All are inherited syndromes of defective DNA repair, except :
A.Xeroderma pigmentosa
B.Sturge weber syndrome
C.Ataxia telangiectasia
D.Bloom syndrome
ANS B

67. Benign cystic taratoma may undergo the following malignancy, except :
(A)Adenocarcinoma
(B)Squamous cell carcinoma
(C)Melanoma
(D)Thyroid carcinoma
Ans a
Squamous cell carcinoma (SCC) is the most common form of malignant transformation in mature cystic teratoma (MCT) of the ovary
melanoma CAN ARISE in a mature cystic teratoma subsequently metastasizing to the lungs
Thyroid carcinoma CAN originate in a teratoma of the ovary

68. The following conditions help in development of sickling, except :
(A) Higher MCHC
(B) A decrease in pH of blood
(C) Increased length of time of low oxygen tension
(D)An increase in pH of blood
Ans d

69. The following are hereditary syndromes of G.I. Tract, expect :
(A)Cowden disease
(B)Familial adenomatous polyposis
(C)Garden syndrome
(D)Bloom syndrome
Ans d
• Cowden disease (CD), also termed Cowden syndrome and multiple hamartoma syndrome, is an autosomal dominant condition with variable expression that results most commonly (80%) from a mutation in the PTEN gene on arm 10q, as reported by Liaw et al. A broader category, "PTEN (phosphatase and tensin homolog) hamartoma tumor syndrome," has been suggested as a name to combine multiple phenotypic presentations all due to PTEN genetic diseases. Rare cases of CD are due to a germline mutation in BMPR1A (bone morphogenetic proteins).
• CD causes hamartomatous neoplasms of the skin and mucosa, GI tract, bones, CNS, eyes, and genitourinary tract.
• Skin is involved in 90-100% of cases, and the thyroid is involved in 66% of cases
GI tract abnormalities in Cowden syndrome
• GI abnormalities are present in as many as 72% of patients.
• Polyps can occur in the esophagus, stomach, small or large intestine, or anus and are most common in the colon. Although Chen et al reported a few cases of adenocarcinoma of the colon in CD patients, the malignant potential of polyps is low. Esophageal glycogen acanthosis has been documented in several patients with CD
• Bloom syndrome is characterized by short stature and a facial rash that develops shortly after first exposure to sun. This rash can make a butterfly-shaped patch of reddened skin on the cheeks. The rash can develop on other sun-exposed areas such as the backs of the hands. Other clinical features include (1) a high-pitched voice; (2) a distinct facies, including a long, narrow face, a small lower jaw, and prominent nose and ears; (3) other pigmentation changes of the skin, including hypo- and hyper-pigmented areas and "cafe-au-lait" spots; (4) telangiectasias (dilated blood vessels) which can appear on the skin but also in the eyes; (5) a moderate immune deficiency, characterized by deficiency in certain immunoglobulin classes, that apparently leads to recurrent pneumonia and ear infections; (6) hypo-gonadism characerized by a failure to produce sperm, hence infertility in males, and premature cessation of menses (premature menopause), hence sub-fertility in females. However, several women with Bloom syndrome have had children.
• Complications of the disorder may include chronic lung problems, diabetes, and learning disabilities. In a small number of persons, there is mental retardation. The most striking complication of the disorder is susceptibility to cancer, as described in more detail in the next section.

70. Prostaglandins in acute inflammation will cause all, except :
(A)Vasodilatation
(B)Pain
(C)Fever
(D)Tissue damage
A

Vasodilatation and inhibition of mediator release represent two distinct mechanisms for prostaglandin modulation of acute mast cell-dependent inflammation
• Prostaglandins Eicosanoid Mast cells
– A group of lipids which can cause vasodilation, fever, and pain.

71. All the following microbiological agents will cause myocarditis, except :
(A)Cytomegalovirus
(B)Candida
(C)Meningococcus
(D)Falciparum malariae
(E)All the above
(F)
All of them cause
Causes of Myocarditis
• Viral - Enterovirus, coxsackie B, adenovirus, influenza, cytomegalovirus, poliomyelitis, Epstein-Barr virus, HIV-1, viral hepatitis, mumps, rubeola, varicella, variola/vaccinia, arbovirus, respiratory syncytial virus, herpes simplex virus, yellow fever virus, rabies, parvovirus
• Rickettsial - Scrub typhus, Rocky Mountain spotted fever, Q fever

• Bacterial - Diphtheria, tuberculosis, streptococci, meningococci, brucellosis, clostridia, staphylococci, melioidosis, Mycoplasma pneumoniae, psittacosis
• Spirochetal - Syphilis, leptospirosis/Weil disease, relapsing fever/Borrelia, Lyme disease
• Fungal - Candidiasis, aspergillosis, cryptococcosis, histoplasmosis, actinomycosis, blastomycosis, coccidioidomycosis, mucormycosis
• Protozoal - Chagas disease, toxoplasmosis, trypanosomiasis, malaria, leishmaniasis, balantidiasis, sarcosporidiosis
• Helminthic - Trichinosis, echinococcosis, schistosomiasis, heterophyiasis, cysticercosis, visceral larva migrans, filariasis
• Bites/stings - Scorpion venom, snake venom, black widow spider venom, wasp venom, tick paralysis
Drugs (usually causing hypersensitivity myocarditis)
• Chemotherapeutic drugs - Doxorubicin and anthracyclines, streptomycin, cyclophosphamide, interleukin-2, anti-HER-2 receptor antibody/Herceptin
• Antibiotics - Penicillin, chloramphenicol, sulfonamides
• Antihypertensive drugs - Methyldopa, spironolactone
• Antiseizure drugs - Phenytoin, carbamazepine
• Amphetamines, cocaine, catecholamines

72. In Wegener granulomatosis we find :
(A) Crescentic glomerulonephritis
(B) Minimal change *
(C) Membranoproliferative glomerulonephritis
(D)Chronic glomerulonephritis
Ans a
In Wegener granulomatosis kidney biopsy shows acute necrotizing and pauci-immune acute crescentic glomerulonephritis

73. "Schiller-Duval body" seen is :
(A)Androblastoma
(B)Choriocarcinnoma
(C)Endodermal sinus (yolk-sac) Tumor
(D)Dysgenninoma
Ans c

74. Which of the following cell is a phagocyte ?
(A)Lymphocyte
(B)Neutrophil
(C)Eosinophil
(D)Basophil
Ans b
Neutrophils are phagocytes, capable of ingesting microorganisms or particles. They can internalise and kill many microbes

75. Commonest cause of SVC syndrome is :
(A)Extrinsic compression
(B)Atherosclerosis
(C)Hamartoma
(D)Mediastinal lymphoma
A,d
• the most common etiology of superior vena cava syndrome is related to malignancy.
• Prior to modern antibiotics, infectious causes including syphilis, tuberculosis, and fungi occurred with almost equal frequency.
• The most common cause of malignancy-related SVCS is bronchogenic carcinoma, which accounts for nearly 80% of cases.
• Lymphoma accounts for approximately 15% of cases.
• Other cases have a variety of causes, including infectious and catheter-related etiologies. Increasingly, dialysis catheters and pacemaker leads are becoming associated with SVCS due to thrombosis
• Superior vena cava syndrome (SVCS), or superior vena cava obstruction (SVCO), is usually the result of the direct obstruction of the superior vena cava by malignancies- such as compression of the vessel wall by right upper lobe tumors and/or mediastinal lymphadenopathy.
– The most common malignancies that cause SVCS is non small cell lung cancer and small-cell lung cancer. It can also occur as a result of thrombosis in the SVC, although this is less common (approximately 35% due to the use of intravascular devices).

76. Single file appearance of cell is seen in :
(A)Invasive lobular carcinoma
(B)Invasive ductal carcinoma
(C)Medullary carcinoma
(D)Tubular carcinoma
Ans a
• Infiltrating lobular carcinoma of the breast, which constitutes 7-15% of invasive breast cancers, presents a diagnostic challenge because of its variable presentation on imaging and clinical examination. I
– t is postulated that the histologic characteristics of infiltrating lobular carcinoma are responsible for the imaging difficulties.
– Typically these tumors show a single-file infiltration of malignant cells through the breast stroma with a relative paucity of desmoplastic response, hemorrhage, necrosis, or calcification
Invasive lobular carcinoma, grade II/III, with cells exhibiting moderate nuclear pleomorphism but maintaining a single file growth pattern

77. Lacunar cell predominantly seen in :
(A)Nodular sclerosis
(B)Mixed cellularity
(C)Lymphocyte Predominant
(D)Lymphocyte depletion
Ans a
Lacunar cell in nodular sclerosis Hodgkin’s lymphoma

78. Gandy gamma bodies are seen in :
(A)Congestive splenomegaly
(B)Infective endocarditis
(C)Rheumatic heart disease
(D)Sarcoidosis
Ans a
• Also known as Gamna Gandy nodules, siderotic nodules and fibrosiderotic nodules, the lesions shown here are composed of fibrous tissue and elastic fibers with deposition of iron and calcium salts and completely disrupt the local architecture of the spleen. They are seen in cases of congestive splenomegaly, sickle cell anemia, and hemochromatosis. Grossly, small yellow-brown, rusty nodules are seen.

79. Onion-skin appearance of bone-tumor in X-ray is seen in :
(A)Chronic osteomyelitis
(B)Ewing's sarcoma
(C)Osteogenic sarcoma
(D)Giant cell tumor of bone
Ans b
Ewing sarcoma
Common manifestations on conventional radiography include
• Poorly marginated, lytic, destructive lesion
Permeative (small holes) or moth-eaten (mottled) appearance
Rarely, they can be sclerotic
• Soft tissue mass or infiltration is common
Soft tissue mass may occur without destruction of cortex
Soft tissue mass may produce saucerization (scalloped depression in cortex)
• Osteosclerosis may be present secondary to reactive bone formation
• Periosteal reaction is common
Lamellated - onion-skinning due to successive layers of periosteal development
Sunburst or spiculated - hair-on-end appearance when new bone is laid down perpendicular to cortex along Sharpey’s fibers
Codman’s triangle - formed between elevated periosteum with central destruction of cortex

80. Acanthosis Nigricans is associated with all, except :
(A)Gastric carcinoma
(B)Lung carcinoma
(C)Uterine carcinoma
(D)Thyroid carcinoma
Ans c
Acanthosis nigricans -Causes
• Crouzon craniofacial dysostosis
• Stomach cancer
• Diabetes mellitus type 2
• Melanoma
• Familial lipodystrophy of limbs and trunk
• Insulin receptor defect with insulin-resistant diabetes mellitus
• Beare-Stevenson cutis gyrata syndrome
• Rabson-Mendenhall syndrome
• Berardinelli lipodystrophy syndrome
• Polycystic ovary syndrome
• Leprechaunism
Acanthosis Nigricans Associated With Lung Squamous Cell Carcinoma , adenocarcinoma of the thyroid gland.

81. are true about placental site Trophoblastic tumor, except :
(A) Trophoblast deeply invades myometrium
(B) They have prominent cytotrophoblastic elements
(C) The trophoblastic cells are immunoreactive for placental lactogen
(D) About 10% have disseminated metastasis
Ans b
• Gestational trophoblastic disease is a spectrum of trophoblastic tumors including hydatidiform mole (partial and complete), invasive mole, gestational choriocarcinoma, and placental site trophoblastic tumor
• It is characterized by abnormal proliferation of pregnancy-associated trophoblastic tissue with malignant potential, which can occur after a normal pregnancy, abortion, ectopic pregnancy, or molar pregnancy. The presence and course of the disease can be monitored with quantitative levels of human chorionic gonadotropin (HCG).
• Placental site trophoblastic tumor is the least common form of gestational trophoblastic disease and is potentially malignant
• Patients usually present with vaginal bleeding 1 week to 14 years after pregnancy.
• The tumor is composed of intermediate trophoblastic cells that normally play a critical role in implantation. In addition, the tumor contains little syncytiotrophoblastic tissue; thus, the HCG levels are usually normal to mildly elevated
• The PSTT was composed primarily of intermediate trophoblasts that contained prominent paranuclear filaments not seen in the intermediate trophoblasts of the choriocarcinomas.
• Rare cells resembling syncytiotrophoblasts were found in the PSTT, but no cytotrophoblasts were observed
• The minimally elevated ß-HCG level that does not increase with serial determinations can help differentiate placental trophoblastic tumor from other types of gestational trophoblastic disease.
• Placental-site trophoblastic tumor should be suspected when the ß-HCG level is minimally elevated and there is either a decrease or no change of endometrial or myometrial mass or ß-HCG level on follow-up studies.
• The diagnosis is made on the basis of findings at biopsy. Ten to fifteen percent metastasize, usually to the lungs, liver, abdominal cavity, and brain.
38-year-old woman (gravida 1, para 0) with history of blighted ovum who presented with intermittent pelvic pain and vaginal bleeding of 3 months’ duration after dilatation and curettage.
Sonograms of pelvis 1 month (A and B) and 2 months (C and D) after dilatation and curettage procedure was performed for blighted ovum show stable vascular mass in lower uterine segment (arrows
Axial fast spin-echo MR images show mass of heterogeneous signal in endometrial cavity with disruption of normal low-signal junctional zone (arrows).
Unenhanced and gadolinium-enhanced gradient echo images show hypervascular endometrial mass with extension into myometrium (arrows
Spin-echo gadolinium-enhanced MR images show myometrial invasion (arrows).
Placental site trophoblastic tumor: Human placental lactogen and pregnancy-associated major basic protein as immunohistologic markers

82. these are inflammatory blistering disease, except :
(A)Bullous pemphigoid
(B)Porphyria
(C)Pemphigus vulgaris
(D)Pemphigus erythematosis
Ans b
Bullous pemphigoid, an autoimmune and inflammatory skin blistering disease
Pemphigus vulgaris - inflammatory skin blistering disease

83. Facioscapulohumoral mucular dystrophy is a disease of nature :
(A)Autosomal dominant
(B)Autosomal recessive
(C)X-linked recessive
(D)X-linked dominant
Ans a
• Facioscapulohumeral muscular dystrophy (FSHMD, FSHD or FSH), which is also known as Landouzy-Dejerine, is an autosomal dominant form of muscular dystrophy that initially affects the skeletal muscles of the face (facio), scapula (scapulo) and upper arms (humeral).

84. Clinical criteria producing clue regarding nature of Thyroid nodule include all,except :
(A)Solitary nodule likely to be neoplastic
(B)Nodule in younger patient likely neoplastic
(C)Nodule in female likely neoplastic
(D)"Hot nodule" in radioactive emaging likely to be benign
Ans c
Factors suggesting a malignant diagnosis include the following:
• Age younger than 20 years or older than 70 years
• Male sex
• Associated symptoms of dysphagia or dysphonia
• History of neck irradiation
• Prior history of thyroid carcinoma
• Firm, hard, or immobile nodule
• Presence of cervical lymphadenopathy
Factors suggesting a benign diagnosis include the following:
• Family history of autoimmune disease (eg, Hashimoto thyroiditis)
• Family history of benign thyroid nodule or goiter
• Presence of thyroid hormonal dysfunction (eg, hypothyroidism, hyperthyroidism)
• Pain or tenderness associated with nodule
• Soft, smooth, and mobile nodule
Thyroid scintigraphy
• In most centers, the routine initial diagnostic evaluation of a solitary thyroid nodule no longer includes imaging studies. In the past, radionuclide scanning was an important imaging study performed routinely in the initial assessment of a thyroid nodule. Nuclear imaging can be used to describe a nodule as being hot, warm, or cold on the basis of its relative uptake of radioactive isotope.
• Hot nodules indicate autonomously functioning nodules, warm nodules suggest normal thyroid function, and cold nodules indicate hypofunctional or nonfunctional thyroid tissue.
• Hot nodules are rarely malignant; however, 5-8% of warm or cold nodules are malignant

85. Patients with sickle cell anemia having pyogenic osteomyelitis, the commonest pathogen is :
(A)Staphylococcous aureus
(B)H. influenza
(C)Pseudomonas
(D)Salmonella
Ans d
• seventy percent of all lesions or blood cultures in children with hemoglobinopathies and presumed osteomyelitis yield Salmonella microorganisms, 10% contain S aureus, and aerobic Gram-negative rods are isolated in 7%

86. All these are true about Legionella disease, except :
(A) Caused by Legionella Pneumophila
(B) The pneumonia is multifocal
(C) The inflammation is fibrinopurulent
(D)Proximal bronchi and bronchioles are involved
Ans d
• Pneumonia is the predominant clinical syndrome of Legionnaires disease. Chest radiography findings vary and are nonspecific and indistinguishable from those observed with other pneumonias.
• Although initial chest radiography findings may be normal, especially in patients with nosocomial disease, the usual progression of findings on serial studies is from patchy areas or nodular appearance to multilobar, almost homogeneous, infiltrates.
• Unilateral involvement is more common than bilateral involvement. Purely interstitial infiltrates are rare. Pleural effusion, present in at least one third of patients, may be the only abnormality. In adults, cavitation is more common in patients who are immunocompromised but has been described both in immunocompromised and immunocompetent children.21, 22, 23
• Radiographic findings usually progress despite appropriate antibiotic therapy; infiltrates may take as long as 4 months to completely resolve. Permanent bullous emphysema requiring lobectomy has been reported
Histologic description of lung tissue in the acute
phase of infection with Legionella defines a fibrinopurulent process with both alveolar and bronchiolar exudates cornposed of neutrophils, macrophages, fibrin, proteinaceous material, and red blood cells.ý Features of acute diffuse alveolar damage may be present. Late sequelae of bronchiolar obliteration and alveolar fibrosis have also been reported\
Chest roentgenogram showing bilateral lower and midzone infiltration with right-sided pleural effusion during acute pneumonia.
Section ofleft upperlobe lung biopsy specimen showing diffuse infiltration of mononuclear
cells (hematoxylin-eosin, original magnification x 100). B, right. Detail ofsame area identifying monotypic
infiltratingcells as plasmacells and demonstratingthe absence ofgranulocytes and otheracute inflammatory
cells (original magnification x 400).

87. All these are true about metastatic calcification, except :
(A) There is hypercalcaemia
(B) Seen in hyperparathyroidism
(C) Seen in Monckebergs sclerosis
(D) Calcium salt deposit in normal tissue
A

Metastatic calcification is deposition of calcium salts in otherwise normal tissue, because of elevated serum levels of calcium in blood, which can occur because of deranged metabolism, synthesis or disposal of calcium and related minerals.

88. In ealry gastric carcinoma all are true, except :
(A)Mucosa is involved
(B)Submucosa is involved
(C)Perigastric lymph node may be involved
(D)It is synonymous with carcinoma-in-situ
Ans d
- Early gastric carcinoma is defined as carcinoma confined to the mucosa or submucosa irrespective of lymphnode involvement (corresponds to T1 gastric carcinoma).
- Macroscopic subypes
• Often a combination of these three types are present.
- Depressed Type (IIC) is the commonest form of early gastric carcinoma.
- Histological features are similar to those of advanced carcinoma. Classified according to Lauren’s Classification into : intestinal, diffuse (signet ring cell) or mixed types.
- Type I and IIa : usually multiple lesions and are likely to be well-differentiated (intestinal type)
- Type IIc & III: Poorly differentiated or signet-ring cell type.
- Tumour behaviour can be predicted by the shape of advancing edge of tumour through the muscularis mucosae into the submucosa.
Early gastric carcinoma with a broad pushing edge (PEN A -subtype) has a poor prognosis.
Tumour with a sharp infiltrating edge (PEN B - subtype) has a better prognosis.
• lymph node metastasis was found in mucosal tumor smaller than 1 cm, and anatomic N1 (perigastric lymph node) lymph node was involved only when tumor size was larger than 1 cm.
• Anatomic N1 plus N2 but limited in No. 7 and 8a (left gastric artery and common hepatic artery) lymph node metastases were found in submucosal tumors.

89. Which tetracycline can be used safely in presence of renal impairment ?
(A)Oxytetracycline
(B)Chlortetracyline
(C)Doxycycline
(D)Tetracycline
C

• Doxycycline has been considered a safe broad-spectrum antibiotic for patients with renal failure

90. Which one is preferred for the treatment of neurocysticercosis ?
(A)Mebendazole
(B)Albendazole
(C)Thiabendazole
(D)Praziquantel
Ans b
• Drug therapy for neurocysticercosis is controversial. Several nonrandomized case series have suggested faster resolution of lesions with treatment using both praziquantel and albendazole.6 Other controlled, randomized trials have not shown such a beneficial effect in intraparenchymal CNS cysticerci. Furthermore, anthelmintic therapy may exacerbate obstruction of CSF flow, precipitating hydrocephalus. Inflammatory response associated with drug therapy may impair vision in ocular disease and may increase the risk of paralysis with spinal cord lesions.
• Praziquantel
– More expensive than albendazole and probably less effective. Available in 600-mg tabs. Increases cell membrane permeability in susceptible worms, resulting in a loss of intracellular calcium, massive contractions, and paralysis of their musculature. In addition, produces vacuolization and disintegration of schistosome tegument. This is followed by attachment of phagocytes to parasite and

91. Which drug follows both first-order and zero-order kinetics for elimination at different doses ?
(A)Phenytoin
(B)Theophylline
(C)Both (A) and (B)
(D)Penicillin G.
C

Most antiepileptic drugs follow first-order kinetics, whereas phenytoin follows zero-order
• Phenytoin metabolism is dose dependent. Elimination follows first-order kinetics (fixed percentage of drug metabolized during a per unit time) at the low drug concentrations and zero-order kinetics (fixed amount of drug metabolized per unit time) at higher drug concentrations.
• This change in kinetics reflects the saturation of metabolic pathways. Thus, very small increments in dosage may result in adverse effects.
The incidence of theophylline side effects is difficult to asses* but is estimated at an overall rate of 21% in patients taking iheophylline (66). There is significant overlap of the therapeutic and toxic scrum ranges for theophyllinc. As a result, symptoms of toxicity can be seen at levels within the conventional therapeutic window of 10-20 jig/mL. In 1995, the American Association of Poison Comrol Centers reported 2338 case of theoph-ylline toxicity, with 19 fatalities At therapeutic dosage, drug levels follow first-order kinetics in which a constant fraction of drug is eliminated per unit of lime. In the high therapeutic and toxic ranges, mixed first- and zero-order kinetics can occur resulting in lengthened half-lives. Zero-order kinetics occurs when normal mechanisms of elimination are saturated and rather than eliminate a constant fraction of total drug, a constant amount of drug is excreted. This amount may be a progressively smaller fraction of the total drug, resulting in decreased elimination over time (II). Hall-lives of up to 20 hr have been reported. It is important to recall those factors that may lead to alterations in the metabolism of theophyl-line and closely follow scrum levels in their presence.

92. Drug of choice in Paroxysmal Supra Ventricular Tachycardia is :
(A)Quinidine
(B)Verapamil
(C)Amlodipine
(D)Adenosine
D

93. All are immunosuppressive agent, except :
(A)Cyclosporine
(B)Levamisole
(C)Corticosteroids
(D)Tacrolimus
B

Levamisole as an immunostimulant

94. Which one is the wrong statement about Carbamazepine ?
(A)It is an Iminostilbene derivative
(B)Effective in grand mal epilepsy and trigeminal neuralgia
(C)Found to be effective in manic-depressive psychosis
(D)Useful in absence seizure
D

• Carbamazepine (CBZ) is an anticonvulsant and mood stabilizing drug used primarily in the treatment of epilepsy and bipolar disorder.
• It is also used to treat ADD, ADHD, schizophrenia, phantom limb syndrome, paroxysmal extreme pain disorder, and trigeminal neuralgia.
• Carbamazepine is effective as monotherapy or in combination with other agents for the treatment of partial seizures (particularly complex partial seizures), and generalized tonic-clonic seizures.
– It is ineffective in the treatment of atonic and myoclonic seizures.
– It is ineffective in the treatment of absence seizures, and may exacerbate absence seizures in some patients

95. Drug of choice in drug-induced Parkinsonism is :
(A)Selegiline
(B)Entacapone
(C)Benztropine mesylate
(D)Tacrine
C

Benzatropine mesilate benztropine mesylate, or benztropine marketed as Cogentin, is an anticholinergic drug principally used for the treatment of:
– Drug-induced parkinsonism, akathisia and acute dystonia;
– Parkinson disease; and
– Idiopathic or secondary dystonia.
• Benzatropine is a centrally acting anticholinergic agent with antihistaminic properties resulting from the combination of the tropine portion of the atropine molecule and the benzohydryl portion of diphenhydramine. Animal studies have indicated that anticholinergic activity of benzatropine is approximately one-half that of atropine, while antihistaminic activity approaches that of pyrilamine. Its anticholinergic effects have been established as therapeutically significant in the management of parkinsonism. Benzatropine antagonises the effect of acetylcholine, decreasing the imbalance between the neurotransmitters acetylcholine and dopamine, which may improve the symptoms of early Parkinson’s disease

96. One of the following drugs can be administered along with fatty food :
A.Tetracycline
B.Griseofulvin
C.Captopril
D.Rifampicin
Ans b
• The absorption of griseofulvin from the GIT is irregular.
– Griseofulvin is ineffective when applied locally. Presence of fatty food in the GIT increases its absorption. The plasma t½ is 24 hrs. However, as it gets bound to the Keratin, it is retained for weeks in the skin.

97. All are Prodrug, except :
(A)Levodopa
(B)Omeprazole
(C)Lisinopril
(D)Alpha - methyldopa
C

• Unlike other ACE inhibitors, lisinopril is not a prodrug.

98. Antiarrhythimc drug useful in ventricular arrhythmia is :
(A)Lignocaine
(B)Esmolol
(C)Verapamil
(D)Digoxin
A

99. Topically useful carbonic anhydrase inhibitor in glaucoma is :
(A)Dorzolamide
(B)Acetazolamide
(C)Brinzolamide
(D)Both A and C
Ans d
• Dorzolamide is a topical carbonic anhydrase inhibitor that reduces intraocular pressure by approximately 3.5 to 6mm Hg in patients with open-angle glaucoma
• Topically Administered Dorzolamide and brinzolamide are topical carbonic anhydrase inhibitors

100. Diuretic of choice in cirrhotic oedema is :
(A)Thiazides
(B)Spironolactone
(C)Amiloride
(D)Urea
B

The major reason for sodium retention is hyperaldosteronism in cirrhostic patients, due to increased activity of the renin - angiotension system. There is avid reabsorption of sodium from the distal tubule and collecting duct.
Diuretics can be divided into two main groups according to their site of action. The first group inhibit Na+ - K+ - 2 CI co transporter in the ascending limb of the loop of henle and include frusemide (furosemide) and bumetamide. It is not appropriate to use these alone since tghe sodium remaining in the tubule sas a result of diuretic action is reabsorbed in the distal tubule and collecting duct because of hyperaldosteronism. A randomized controlled trial has shown frusemide alone to be less effective than spironolactone. Thiazides inhibit sodium in the distal convoluted tubule, have a longer half - life, and are not as a rule used in the treatment of ascites.
The second group, spironolactone (an aldosterone antagonist), amiloride and triamtcrcne (inhibitors of the Na* channel) block sodium reabsorption in the distal tubule and collecting duct. They are central to the treatment of cirrhotic ascites. They are weakly natriuretic but conserve potassium. Potassium supplements are not usually necessary —indeed this type of diuretic sometimes needs to be temporarily stopped because of hyper kalaemia.
There are two therapeutic approaches which can be used initially: spironolactone alone, or a combination of spironolactone with frusemide. Both have their advocates
Spirunoiactone alone. The starting dose is 100-200 mg/day according to the degree of ascites. If there has been insufficient clinical response (less than 0.5kg/day weight loss) after 3-4 days, then the dose is increased by 100 mg/day every 4 days to a maximum of 400 mg/day. Lack of clinical response indicates the need to check the urinary sodium output, because a high value will identify the occasional patient who is exceeding the prescribed low sodium diet.
If there is a lack of, or insufficient, clinical response on spironulactone alone (usually at the level of 200 mg/day) a loop diuretic such as frusemide is added at a dose of 20-40 mg/day.

Combination therapy. Treatment is started with the combination of spironolactone (lOOmg) and frusemide (40 mg) daily 65]. There is no direct comparison between this and the use of spironolactone alone. The ease of control and choice of diuretics can be related
to the 24-h urinary sodium content on admission tohospital The disadvantage of starting with spironolactone alone is the delay before its clinical effect.
Monitoring of daily weight is necessary. The rate of ascitic fluid reabsorption is limited to 700-900ml/day. If a diuresis of 2-3 litre is induced, much of the fluid must come from non-ascitic, extra-cellular fluids including oedema fluid and the intravenous compartment- This is safe so long as oedema persists. Indeed diuresis may be rapid (greater than 2kg daily) until oedema disappears [60]. Overall recommendations, however, to avoid the risk of renal dysfunction are a maximum daily weight loss of 0.5 kg/day, with a maximum of 1.0 kg/day in those with oedema.
lntravascular volume expansion with intravenous albumin increases the naturesis in response to diuretics, but is expensive and not cost-effective [20).
Long-term spironolactone causes painful gynaeco-mastia in cirrhotic males and should then be replaced by 10-15mg/day of amiloride. However, this is less effective than spironolactone.
If stopping the offending drug is not possible, then anticholinergic drugs may be used. However, these should not be used in elderly people because they may cause confusion as well as worsening tardive dyskinesia.
Atnantadine (Symmetrei), another drug used to treat Parkinson's, can also be used to treat drug-induced parkinsonism if the person cannot stop the offending drug. However, like antichoJinergic drugs, amantadine also causes confusion and sometimes psychosis in older people, and therefore is only really suitable for younger people with drua-induced parkinsonism.

101. All are microsomal enzyme inhibitors, except
(A)Erythromycin
(B)Clarithromycin
(C)Azithromycin
(D)Ciprofloxacin
C

Ciprofloxacin is an inhibitor of human cytochrome P450 1A2. (CYP1A2) mediated metabolism.
Azithromycin, unlike erythromycin and clarithromycin, is not known to inhibit the cytochrome P450 enzyme system

102. Shortest acting beta – blocker is :
(A)Propranolol
(B)Atenolol
(C)Nadolol
(D)Esmolol
D

103. Safe antihypertensive during pregnancy is :
A.Enalapril
B.Hydrallazine
C.Methyldopa
D.Both B and C
D
• use of hydrallazine parenterally is in the control of hypertensive emergencies, particularly associated with pre-eclampsia and toxae- mia of pregnancy
104. All of these drugs can cause hypokalaemia, except :
(A)Epinephrine
(B)Ramipril
(C)Acetazolamide
(D)Amphotericin B,
B
Causes of Hypokalemia
• Osmotic diuresis: Mannitol and hyperglycemia can cause osmotic diuresis.
• Increased gastrointestinal losses: Losses can result from diarrhea or small intestine drainage.
• Diuretics (carbonic anhydrase inhibitors, loop diuretics, thiazide diuretics): Increased collecting duct permeability or increased gradient for potassium secretion can result in losses.
• Some penicillins
• Exogenous bicarbonate ingestion
• Amphotericin B
• Gentamicin
• Cisplatin
• Osmotic diuresis: Mannitol and hyperglycemia can cause osmotic diuresis.
• Increased gastrointestinal losses: Losses can result from diarrhea or small intestine drainage.
• Diuretics (carbonic anhydrase inhibitors, loop diuretics, thiazide diuretics): Increased collecting duct permeability or increased gradient for potassium secretion can result in losses.
• Some penicillins
• Exogenous bicarbonate ingestion
• Amphotericin B
• Gentamicin
• Cisplatin

105. Which one is the wrong statement about mannitol ?
(A) Increases plasma osmolality
(B) Increases viscosity of blood
(C) Primary site of action is Loop of Henle
(D) Useful in dialysis disequilibrium syndrome
C
Dialysis disequilibrium syndrome (DDS
• The classic DDS refers to acute symptoms developing during or immediately after hemodialysis. Early findings include headache, nausea, disorientation, restlessness, blurred vision, and asterixis. More severely affected patients progress to confusion, seizures, coma, and even death. It is now recognized, however, that many milder signs and symptoms associated with dialysis ­ such as muscle cramps, anorexia, and dizziness developing near the end of a dialysis treatment ­ are also part of this syndrome. The incidence of DDS varies according to the patient population and the attention paid to the preventive measures described below. Severe DDS is now rare in adults because of the standard use of the preventive recommendations made below. However, children may remain at increased risk. A retrospective analysis of 180 children and adolescents on maintenance dialysis foundthat 13 (7 percent) had dialysis-associated seizures. All but one of these patients were treated with hemodialysis. The development of the above symptoms during dialysis is strongly suggestive of DDS. Nevertheless, there are a number of other disorders that must be excluded including uremia itself, subdural hematoma, metabolic disturbances (hyponatremia, hypoglycemia), and drug-induced encephalopathy. Drug accumulation is a particular problem in renal failure with drugs that are normally excreted by the kidney
• the dialysis disequilibrium syndrome (DDS) is a central nervous system disorder that remains an important clinical problem in dialysis patients. It is characterized by neurologic symptoms of varying severity that are thought to be due primarily to cerebral edema. New patients just being started on hemodialysis are at greatest risk, particularly if the BUN is markedly elevated (above 175 mg/dL or 60 mmol/L). Other predisposing factors include severe metabolic acidosis, older age, pediatric patients, and the presence of other central nervous system disease such as a preexisting seizure disorder.
• Prevention is the mainstay of therapy in the DDS, particularly in new dialysis patients who are at highest risk. The initial dialyses should be gentle, but repeated frequently. The aim is a gradual reduction in BUN, which will be protective but may not prevent mild symptoms such as headache and malaise. Slow urea removal can be achieved by one of the following methods: With hemodialysis, therapy can be initiated with two hours of dialysis at a relatively low blood flow rate of 150 to 250 mL/min with a small surface area dialyzer. This regimen, which is repeated daily for three or four days, is different from the standard every other day four-hour regimen at high flow rates. If the patient shows no signs of DDS, the blood flow rate can be increased by 50 mL/min per treatment (up to 300 to 400 mL/min) and the duration of dialysis can be increased in 30 minute increments (up to four or more hours, as necessary for adequate solute removal). Patients who also have marked fluid overload can be treated with ultrafiltration (which removes less urea per unit time) followed by a short period of hemodialysis.
• The patient can be started on peritoneal dialysis in which the low rate of peritoneal blood flow results in a urea clearance per unit time that is much lower than that with hemodialysis. The DDS has not been reported with continuous peritoneal dialysis. Some physicians recommend prophylactic phenytoin (1000 mg loading dose followed by 300 mg/day until uremia is controlled) and/or the administration of 12.5 g of hypertonic mannitol intravenously every hour of dialysis in high-risk patients with marked azotemia (BUN above 150 to 200 mg/dL [54 to 71 mmol/L]) or an underlying alteration in mental status. Symptoms of DDS are self-limited and usually dissipate within several hours. Severe DDS with seizures can be reversed more rapidly by raising the plasma osmolality with either 5 mL of 23 percent saline or 12.5 g of hypertonic mannitol.

106. Corticosteroid with maximum mineralocorticoid activity is :
(A)Prednisolone
(B)Fludrocortisone
(C)Triamcinolone
(D)Hydrocortisone
B

107. Antihyperglycemic but not hypoglycemic drug used in diabetes mellitus include
(A)Metformin
(B)Acarbose
(C)Tolbutamide
(D)Both (A) and (B)
D
• Acarbose does not produce hypoglycemia; however, hypoglycemia may develop if used together with sulfonylureas or insulin

108. Safe antimalarial drug during pregnancy is :
(A)Primaquine
(B)Chloroquine
(C)Artemisinin
(D)Mefloquine
B

109. Diuretic useful in hypercalcaemia is :
(A)Acetazolamide
(B)Amiloride
(C)Thiazides
(D)Frusemide
D
Hypercalcemia treatment
Initial therapy: fluids and diuretics
• hydration, increasing salt intake, and forced diuresis.
• hydration is needed because many patients are dehydrated due to vomiting or renal defects in concentrating urine.
• increased salt intake also can increase body fluid volume as well as increasing urine sodium excretion, which further increases urinary calcium excretion (In other words, calcium and sodium (salt) are handled in a similar way by the kidney. Anything that causes increased sodium (salt) excretion by the kidney will, en passant, cause increased calcium excretion by the kidney)
• after rehydration, a loop diuretic such as furosemide can be given to permit continued large volume intravenous salt and water replacement while minimizing the risk of blood volume overload and pulmonary edema. In addition, loop diuretics tend to depress renal calcium reabsorption thereby helping to lower blood calcium levels
• can usually decrease serum calcium by 1-3 mg/dL within 24 h
• caution must be taken to prevent potassium or magnesium depletion
Additional therapy: bisphosphonates and calcitonin
bisphosphonates are pyrophosphate analogues with high affinity for bone, especially areas of high bone-turnover.
• they are taken up by osteoclasts and inhibit osteoclastic bone resorption
• current available drugs include (in order of potency): (1st gen) etidronate, (2nd gen) tiludronate, IV pamidronate, alendronate, risedronate, and (3rd gen) zoledronate
• all patients with cancer-associated hypercalcemia should receive treatment with bisphosphonates since the ‘first line’ therapy (above) cannot be continued indefinitely nor is it without risk. Further, even if the ‘first line’ therapy has been effective, it is a virtual certainty that the hypercalcemia will recur in the patient with hypercalcemia of malignancy. Use of bisphoponates in such circumstances, then, becomes both therapeutic and preventative
• patients in renal failure and hypercalcemia should have a risk-benefit analysis before being given bisphosphonates, since they are relatively contraindicated in renal failure.
Calcitonin blocks bone resorption and also increases urinary calcium excretion by inhibiting renal calcium reabsorption
• Usually used in life-threatening hypercalcemia along with rehydration, diuresis, and bisphosphonates
• Helps prevent recurrence of hypercalcemia
• Dose is 4 Units per kg via subcutaneous or intramuscular route every 12 hours, usually not continued indefinitely

110. Interleukins can be produced by :
(A)Monocytes only
(B)Lymphocytes only
(C)Both Monocytes and Lymphocytes
(D)Monocytes, Lymphocytes and many others
D
• interleukins are produced by a wide variety of body cells

111. Accumulation of large excess of body water is followed by :
(A) Osmolarity of urine remains same
(B) Total amount of solute excreted by the kidneys remains same
(C) Urine flow rate remains same
(D) Total amount of urine volume remains same
B
• The BUN:Cr in prerenal azotemia is greater than 20. The reason for this lies in the mechanism of filtration of BUN and creatinine. GFR levels are decreased due to hypoperfusion, leading to a general increase in BUN and creatinine levels. However, since some of the filtered BUN is reabsorbed from the proximal tubules of the kidney back into the blood, whereas very little of the filtered creatinine is, more BUN builds up in the blood relative to creatinine.[2]
• following lab values are important
– 1.BUN:Cr ratio > 20
– 2.low urine sodium <> 500

112.Which one of the following is not essential for the biosynthesis of thyroid hormones ?
(A)Iodine
(B)Ferritin
(C)Thyroglobulin
(D)Thyroid stimulating hormone
B

113.Patients having acute cardiac failure do not show oedema because :
(A)The plasma oncotic pressure is high
(B)There is renal compensation
(C)There is an increase cardiac output
(D)There is a fall in the systemic capillary hydrostatic pressure
B
In congestive heart failure (CHF) impaired cardiac emptying results in a rise in ventricular end - diastolic pressure, the consequent high venus pressure promotes transudation of fluid out of hte vascular channel through several mechanism. First, the mean capillary hydraulic pressure in increased. Because the resulting rate of fluid transfer from peripheral capillaries to the interstitial space exceeds the rate of return of interstitial fluid to the intravascular compartment, edema formation ensure. Furthermore, an increased peripheral venous pressure may cause a neurogenic resetting of precapillary and postcapillary resistances, leading to increased capillary hydrautic pressure and transudation of fluid from the intravascular to the intersitital space. Third, an increase in systemic venous pressure often leads to hepatic congestion, whic may depres both plasma renin clearnance and the hepatic metabolism and aldosterone. Fourth, there is reason to believe that an increase in systemic venous pressure per se may contribute to sodium retention via afferent pathways. There is supportive evidence for the existence of volume receptors in the low - pressure central venous portion of the circulation (referred to as low - pressure volume receptors). It is thus possible that an increased venous pressure activates such receptors, leading to sodium retention by the kidneys. Furthermore salt and water retention in CHF leads to a mild decrease in systemic plasma protein concentration, with a decrease in the oncotic pressure gradient, thereby favoring edema formation. several lines of evidence suggest that the increase in central venous pressure in CHF retards lymph flow in the thoracic duct, with a resultant decrease in lymphatic drainage of pe
The adeptive activation of the renin - angiotension aldosterone (RAA) system seen in CHF, with the resultant increases in circulating and intrarenal angiotensin II level, results not only in better preservation of MAP but in a relative preservation of the GFR in the face of reduced renal perfusion that is, an increase in the filtration fraction (FF), the ratio of GFR/RPF. The increase in FF, which augments GFR for any given level of RPF, is a consequence of the induction of the greater increase of vascular resistance in the efferen than in the afferent arteriole by angiotensin II. This adaptive process minnizes the fall in glomerular capilary hydrostatic pressure that would otherwise occur. In very severe CHF however, a marked increase in intra renal angiotensin II may result in predominant constriction of the afferent arteriole thereby contributing to a fall in GFR.

114. Voltage augmentation in augmented unipolar limb lead is increased by :
(A)1.5 times
(B)2 times
(C)2.5 times
(D)3 times
A
• aVL simply works out to be about 1.5 times the electrical activity recorded by VL

115. Which one of the following is not a function of the limbic system ?
(A)Central autonomic functions
(B)Controls primitive behaviour like sex and hunger
(C)Plays important role in emotion of rage and fear
(D)Control intellectual activities
D
• The limbic system operates by influencing the endocrine system and the autonomic nervous system. It is highly interconnected with the nucleus accumbens, the brain’s pleasure center, which plays a role in sexual arousal and the "high" derived from certain recreational drugs. These responses are heavily modulated by dopaminergic projections from the limbic system. In 1954, Olds and Milner found that rats with metal electrodes implanted into their nucleus accumbens repeatedly pressed a lever activating this region, and did so in preference to eating and drinking, eventually dying of exhaustion.[6]
• The limbic system is also tightly connected to the prefrontal cortex. Some scientists contend that this connection is related to the pleasure obtained from solving problems. To cure severe emotional disorders, this connection was sometimes surgically severed, a procedure of psychosurgery, called a prefrontal lobotomy (this is actually a misnomer). Patients who underwent this procedure often became passive and lacked all motivation.

116. The hypothalamus regulates all the activities, except
(A)Formation of anterior pituitary hormones
(B)Release of releasing factors
(C)Release of inhibitory factors
(D)Control circadian rhythm
A

117. Spatial orientation of nerve fibers from the different parts of the body exists in the dorsal column of medial lemniscal system :
(A)The fibers from the lower part of the body lie laterally
(B)The fibers from the trunk is present centrally
(C)The fibers from the upper part of the body are pushed centrally
(D)The fibers from the lower part of the body lie toward the center of the cord
D

118. Angiotensin II stimulates thirst by a direct action on receptors located in :
(A)Posterior pituitary
(B)Area prostrema
(C)Supraoptic crest
(D)Subfornical organ
D
Neural effects
• Angiotensin II increases thirst sensation (dipsogen) through the subfornical organ (SFO) of the brain, decreases the response of the baroreceptor reflex, and increases the desire for salt.
• It increases secretion of ADH in the posterior pituitary and secretion of ACTH in the anterior pituitary.
• It also potentiates the release of norepinephrine by direct action on postganglionic sympathetic fibers.

119.Vasopressin after being secreted in blood is bound to :
(A)Albumin
(B)Lx Globulin
(C)Neurophysin I
(D)Neurophysin II
D
oxytocin is bound to neurophysin I and vasopressin to neurophysin II

120. Which one of the following is not correct ? Tapping the tendon of a muscle may elicit ?
(A)Contraction of the muscle
(B)Relaxation of the muscle
(C)No response
(D)Fibrillation of the muscle
B

121. happen to blood pH in haernorrhagic shock ?
(A)Acidosis
(B)Alkalosis
(C)No change
(D)Either (A) or (B)
B

122. Increased Gamma efferent discharge is seen in all, except :
(A)Jendrassik's maneuvre
(B)Anxiety
(C)Rapid shallow breathing
(D)Stimulation of skin
C
When the rate of gamma efferent discharge is increased, the intrafusal fibers are shorter than the extrafusal one. If the whole muscle is stretched during the stimulation of the gamma afferents, additional action potential are generated by additional stretch and the rate of discharge in the Ia fibers is further increased. Increased gamma efferent discharge along with the increased discharge of the alpha motor neuron initiates movement. Reflexes may be difficult to elicit in normal subjects owing to global hypoexcitability of ventral horn motor
neurons. Under these circumstances, motor neurons may be rendered more excitable by the Jendrassik maneuver (reinforcement). This was originally thought to enhance fusimotor drive, though it is now thought to reflect a direct excitatory effect on the a motor neurons.
It has been suggested that absent ankle jerks may not be abnormal in the elderly. However, only 6.0% of normal subjects over the age of 65 have absent ankle jerks.13,14 A recent study has focused attention on the technique for eliciting the ankle jerk in elderly people, suggesting that plantar strike is more effective than tendon strike.15
Other factors influencing knee jerk are, anxiety, stimulation of skin with noxious agents, reinforcement maneuvers all increases gamma discharge.
• Jendrassik’s manoeuvre medical dictionary
– A method of emphasizing the patellar reflex: the subject hooks his hands together by the flexed fingers and pulls against them with all his strength.

123. t Absolute refractory period of heart is the gap of time in which :
(A) Heart is in diastole
(B) Responsive to neural stimuli
(C) No action potential from another part of heart will reexcite the heart muscle
(D) None of the above
C
After initiation of an action potential, the refractory period is defined two ways:
• The absolute refractory period is the interval during which a second action potential absolutely cannot be initiated, no matter how large a stimulus is applied.
• The relative refractory period is the interval immediately following during which initiation of a second action potential is inhibited but not impossible.
– The absolute refractory period coincides with nearly the entire duration of the action potential. In neurons, it is caused by the inactivation of the Na+ channels that originally opened to depolarize the membrane. These channels remain inactivated until the membrane repolarizes, after which they close, reactivate and regain their ability to open in response to stimulus.
• The relative refractory period immediately follows the absolute. As voltage-gated potassium channels open to terminate the action potential by repolarizing the membrane, the potassium conductance of the membrane increases dramatically. K+ ions flooding out of the cell bring the membrane potential closer to the equilibrium potential for potassium. This causes brief hyperpolarization of the membrane, that is, the membrane potential becomes transiently more negative than the normal resting potential. Until the potassium conductance returns to the resting value, a greater stimulus will be required to reach the initiation threshold for a second depolarization. The return to the equilibrium resting potential marks the end of the relative refractory period.

124. The intrafusal fibers of striated skeletal muscles are innervated by one of the following type of motor neurons :
(A)Alpha
(B)Beta
(C)Gamma
(D)Delta
C
• Intrafusal muscle fibers are skeletal muscle fibers that comprise the muscle spindle and are innervated by gamma motor neurons. These fibers are a proprioceptor that detect the amount and rate of change of length in a muscle.
• These fibers are walled off from the rest of the muscle by a collagen sheath. This sheath has a spindle or "fusiform" shape, hence the name "intrafusal."
• Intrafusal muscle fibers are not to be confused with extrafusal muscle fibers, which are innervated by alpha motor neurons and contract, generating skeletal movement.
• It is by the sensory information from these two intrafusal fiber types that one is able to judge the position of their muscle, and the rate at which it is changing.

125. Loop of Henle handles the following ions, except :
(A)Na+
(B)K+
(C)Cl-
(D)Urea
D
Loop of Henle

126. A measure location dividing the distribution in the ratio of 1 : 3 is :
(A)Median
(B)First quartile
(C)Third quartile
(D)Mode
B
• A quartile is any of the three values which divide the sorted data set into four equal parts, so that each part represents one fourth of the sampled population.
• In epidemiology the quartiles are the four ranges defined by the three values discussed here.
• first quartile (designated Q1) = lower quartile = cuts off lowest 25% of data = 25th percentile
• second quartile (designated Q2) = median = cuts data set in half = 50th percentile
• third quartile (designated Q3) = upper quartile = cuts off highest 25% of data, or lowest 75% = 75th percentile
– The difference between the upper and lower quartiles is called the interquartile range.
• The location of other points in the range can be measured by Percentiles of centiles, which are values in a series of observations, arranged in ascending order of magnitudes which divides the distribution into 100 equal parts. Median is 50th percentile. i.e. it will have 50% observations on either side, 10th percentile should have 10% observations to the left and 90% of the right. Percentiles are used to divide a distribution into convenient groups. Those in common use are:
• Quartile denotes division of the entire range of distribution into four equal parts, of equal probability by three equidistant points
First Quartile - will have distribution in the ratio = 1:3 (as 25% observations are to the left and 75% to the right)
Second Quartile - will have distribution in ratio = 2:2 (or 1:1 as 50% observations are to the left and 50% to the right)
Third Quartile - will have distribution in the ratio= 3:1 (as 75% of observations are to left and 25% to the right)
Quintiles: Four in number, divide the distribution into equal parts. So 20th percentile or first quintile will have 20% observations falling on its left and 80% to its right.
Deciles: Nine in number, divide the distribution into 10 equal parts. First decile or 10th percentile will have 10% observations on its left and 90% to its right 5th decile will be same as median.
• Explanation
1. Median is the middle value, when all the values are arranged either in ascending or descending order.
2. First quartile will have distribution in the ratio = 1:3 (as 25% observations are to the left and 75% to the right).
3. Third quartile will have distribution in the ratio= 3:1 (as 75% of observations are to left and 25% to the right).
4. Mode is the most commonly occuring value.

127. Quantities continuous data can be presented by the following diagrams, except
(A)Histogram
(B)Frequency polygon
(C)Ogive
(D)Bar chart
D
A histogram is a way of summarising data that are measured on an interval scale (either discrete or continuous
Bar Chart - use with discrete data
• If you are given discrete data you can put it into a bar chart as in the example below. The height of the bars in the chart represent the frequency (no. of times something was recorded
Histogram - use with grouped and continous data
A histogram looks very similar to a bar chart, but it is actually very different.
Instead of the height of the bars representing the frequency it is actually the frequency density or area of the bars. The frequency density is always on the vertical axis ().
• There are no gaps between the bars (indicating that the data is continuous) in the histogram and the width of them can vary (depending on the sizes of the class intervals).
• Frequency density = Frequency / Width of Bar.

128. Chi-square test is used to measure the degree of :
(A)Association between two variables
(B)Correlation between two variables
(C)Strength of association
(D)None of the above
A
We quantify the strength of association using the relative risk and the odds ratio
• When using the chi-square statistic, these coefficients can be helpful in interpreting the relationship between two variables once statistical significance has been established. The logic for using measures of association is as follows:
• Even though a chi-square test may show statistical significance between two variables, the relationship between those variables may not be substantively important. These and many other measures of association are available to help evaluate the relative strength of a statistically significant relationship. In most cases, they are not used in interpreting the data unless the chi-square statistic first shows there is statistical significance (i.e., it doesn’t make sense to say there is a strong relationship between two variables when your statistical test shows this relationship is not statistically significant).

129. To calculate the predictive value of a negatives test, the denominator is composed of :
(A)True Positives + False Negatives
(B)True Positives + False Positives
(C)True Negatives + False Negatives
(D)True Positives + True Negatives
C

130. If the annual risk of tuberculosis infection is 1.8%, then the number of new sputum smear positive cases per 1 lac per year will be :
(A)50
(B)75
(C)90
(D)95

131. Out of new adult patients attending a PHI, the proportion of chest symptomatics requiring sputum smear examination for AFB is about :
(A)2-3%
(B)5%
(C)1%
(D)10%
• cases of pulmonary tuberculosis which are missed by sputum smear examination by microscopy among symptomatic out patients (15% of the total cases)

132. The following are the different types of cases to be placed under category II of DOTS regimen, except :
(A)Relapse
(B)Defaulter
(C)New
(D)Failure
C
Category I which includes, new cases of smear positive pulmonary TB, seriously ill newly diagnosed sputum negative and seriously ill extra-pulmonary cases, is given 2H3R3Z3E3/4H3R3 and
Category II which includes patients who are either smear positive relapse, failure, treatment after default or sub category-others or have already taken treatment for more than one month prior to reporting is treated with 2S3H3R3Z3E3/ 1H3R3Z3E3/5H3R3E3 regimen
• The DOTS Regimen (WHO,1993)
• DOTS is 6 month therapy
• Intensive phase (daily, 2 mo) :
– INH, RMP, PZA, EMB
• Continuation phase (3 times a week, 4 mo):
– INH & RMP

133. The treatment regimen for intensive phase of category III patient under DOTS strategy :
(A)2H3R3E3
(B)2H3R3Z3
(C)2H3R3E3Z3
(D)3H3R3E3
B
• Treatment: WHO short course
Category 1: 2HERZ/4HR
Category 2: 2HERZS/HERZ/5HER
Category 3: Same protocol as category 1
Regimens used were
Cat I: 2H3R3E3Z3/4H3R3,
Cat II: 2S3H3R3E3Z3/1H3R3E3Z3/5H3R3E3
Cat-III: 2H3R3Z3/ 4H3R3.

134. The vector mosquito for J.E. is :
(A)Culex vishnui
(B)Culex fatigans
(C)Anopheles stephensi
(D)Aedes aegypti
A

135. In considering Post Exposure Prophylaxis in HIV, the percutaneous exposure in a person having superficial scratch with a solid needle will be categorized as :
(A)EC1
(B)EC2
(C)EC3
(D)EC4
B
Recommendation of post exposure prophylaxis (PEP) regimens based on exposure and status of source (source NACO guidelines)
a) If EC1 and SC1 PEP may not be required
b) If EC1 and SC 2 or Basic regimen required
EC2 and SC 1
c) If EC 2 and SC 2 Expanded regimen warranted
d) If EC3 and SC 1 or 2 Expanded regimen warranted
e) If both EC and SC are not known, or if EC2 or EC3 in the absence of Sc Basic regimen required
136. "Working together for health" is the WHO slogan for the year :
(A)2005
(B)2006
(C)2007
(D)2008
B
• World Health Day 2006 - Working together for health

137. In the categorization of Biomedical waste, needles and scalpels are placed in :
(A)Cat 1
(B)Cat 2
(C)Cat 3
(D)Cat 4
D

138. In the disposal of injection related wastes at the health facility, the wrapper and cap of AD syringes should be disposed in :
(A)Red Bag
(B)Yellow Bag
(C)Black Bag
(D)Blue Bag
C
• throw the AD syringe wrapper or plastic caps in the black bag

139. In the National Immunization Schedule DT vaccine is administered at the age of:
(A)9 months
(B)5 years
(C)16-24 months
(D)3 years
B

140. Hepatitis B vaccine should be stored at :
(A)-3°C
(B)4°C—8°C
(C)-5°C
(D)0°C
B
• Hep B vaccine is available as single and multidose vials and should be stored at 2 to 8° C. The vaccine should not be frozen; frozen vaccine should be discarded

141. Most heat sensitive vaccine is :
(A)Reconstituted BCG vaccine
(B)Measles
(C)OPV
(D)JE vaccine
C

142. The population of subcentre in a plain area is:
(A)5000
(B)2000
(C)3000
(D)10000
A
• norms of plain areas (5000 per sub-center

143. The following are the principles of Primary Health Care, except :
(A)Intersectorial coordination
(B)Community participation
(C)Advanced technology
(D)Equitable distribution
C
• The Principles of Primary Health Care
• Fair and equitable and timely accessibility to health care providers of choice.
• Public participation in community development.
• Inter-sectoral and transdisciplinary collabortion with equality of all health care team members. • Health Protection, Health Promotion and Illness and Injury Prevention.
• Approriate and affordable Technology, inclusive of information technology

144. Elemental iron and folic acid contents of iron folic acid pediatric tablets supplied under RCH programme are :
(A)20 mg iron and 100 mg folic acid
(B)40 mg iron and 100 mg folic acid
(C)40 mg iron and 50 mg folic acid
(D)60 mg iron and 100 mg folic acid
A
• The daily dosage of Iron Folic Acid (IFA) supplement (20mg elemental iron + 100 mcg folic acid) recommended for children in the current ongoing NNACP was considered appropriate.

145. NPU value of rice is :
(A)60
(B)65
(C)50
(D)80
B
• About 8% of the total energy provided by rice. It is protein of good quality, with net dietary protein (NPU) value of about 65

146. The daily protein requirement of an adult person is (gm/kg) :
(A)1
(B)1.2
(C)1.5
(D)2
A
• The protein RDA is high, to cover most person’s needs. The average requirement for protein is 0.6 grams per kilogram of body weight; the RDA is 0.8 grams this is said to meet 97.5% of the population’s needs

147. Bagassosis occurs due to :
(A)Coal dust
(B)Silica
(C)Sugar cane
(D)Cotton
C
• A pneumoconiosis caused by the inhalation of bagasse dust, a dry sugarcane residue

148. Epidemic typhus is transmitted by :
(A)Rat flea
(B)Louse
(C)Hard tick
(D)Soft tick
B
• Epidemic typhus, spread by the body louse

149. The Type of vector borne transmission in case of Leishmania Donovani is :
A.Propagtive
B.Cyclopropagative
C.Cyclodevelopmental
D.Transovarian
B
Types of Biological Transmission
• Propagative biological transmission, in which the disease-causing organism reproduces in the arthropod, but undergoes no further developmental changes.
– Plague bacillus in flea
– yellow fever virus
• Cyclopropagative biological transmission, in which the disease-producing organism undergoes cylical changes and reproduces in the arthropod.
– Plasmodium in mosquitos
– Trypanosoma in tsetse flies
• Cyclodevelopmental biological transmission, in which the disease-producing organism must undergo development in the arthropod but does not multiply there.
– Microfilarie in mosquitos
• Transovarian transmission, in which disease-causing organisms are transmitted from the infected parent arthropod to their offspring
Leishmania transmission is cyclo - progagative and involves and interactions among the parasite, the vector and a susceptible vertebrate host.
When feeling on infected hosts, female sand flies may ingest amastigotes with the blood meal, some of which may diveide and transform into promastigotes. Leishmanias may be grouped into 3 sections based on recognized pattern of developement in the gut of the sand fly. Parasites in Section hypopylaria are primitive flagellates that develop posteriorly in the pylorus, ileum and rectum. Parasites in this section formerly were classified as Leishmani, but now are placed in a separate genus, sauroleishmania, none are human pathogens. Their only known hosts are old world lizards. because infection in limited to the hindgut of the sand fly, transmission probably occurs when the lizard eats infected sand flies.

150. Coverage Evaluation Survey for Immunization is carried out by which sampling technique :
A.Simple random
B.Stratified Random
C.Lot Quality
D.Systematic random
C
• Lot quality assurance sampling (LQAS)
The sample size for LQAS is based on the hypothesized (or desired) immunization coverage. Keeping rarity of an event of getting an umimmunized child, the sample size was calculated
Lot Quality Assurance Sampling (LQAS) (1)
Lot Quality Assurance Sampling (LQAS) (1)
Origin is from manufacturing
Used to assure a specified level of quality while keeping costs at a minimum
LQAS (2)
Example : Producer of light bulbs. How many need to be tested to assure a specified percentage work? Inversely, to assure that very few fail. Assume testing all light bulbs is too costly and would make the company noncompetitive. How many light bulbs do they to tests?
LQAS (3)
Health example; which immunization clinics are doing a poor job? In each clinic, how many records do you need to review to decide whether the clinic is doing a good job or a poor job?
LQAS (4)
Iodized salt example: IDD was determined a significant national problem. Universal salt iodization is the goal. District health agencies are responsible to assure households in all areas are using iodized salt. When a health official visits a village, how many households need to be visited?
LQAS (5)
N number of samples to take
d threshold value; if more than this number of samples fail, we will reject the lot (i.e., say there is a problem)
d the actual number of samples that fail
LQAS (6)
Pa program goal - the goal of the program
Po the proportion we are willing to accept in order to identify areas with the largest problems
LQAS (7)
Example: National goal, 95% of households using iodized salt. lowest acceptable proportion at the present time, 50%
sample size d value
LQAS (9)
In this example, n = 5 and d = 0 which means
sample 5 households
If any salt samples do not contain adquate iodine levels, then the community "fails, illegal noniodized salt being sold or a problem with iodized salt production.
LQAS (10)
If all salt samples contain adequate ioidine levels, then the community "passes"
Not discussed are issues of hte alpha and beta values used to determine sample sizes.

151.The following statements is true regarding nasal airway :
A less traumatic than oral airway
B less risk of infection compared to oral airway
C better tolerated than oral airway
D can be safely used in anticoagulated patients
C
Indications and contraindications
•Nasopharygeal airways are sometimes used by people who have sleep apnea.
•These devices are also used by emergency care professionals such as EMT'S and paramedics in situations where an artificial form of airway maintenance is necessary but it is impossible or unadvisory to use an oropharyngeal airway, the preferred type of airway adjunct. For example, in a patient having epileptic seizures whose teeth are clenched shut. In an unconscious patient, suction of the upper airways may also be applied via an NPA.
•Insertion of an NPA is contraindicated in patients with severe head or facial injuries, or have evidence of a basal skull fracture (Battle's sign, Raccoon eyes, cerebrospinal fluid/blood from ears, etc.) due to the possibility of direct intrusion upon brain tissue. An oropharyngeal airway may be used instead, but these devices frequently trigger a patient's gag reflex, while nasopharyngeal airways often do not.

1.nasopharyngeal airways are better tolerated than oropharyngeal airways in awake or lightly anesthetized patients

152.All are advantages of nitrous oxide, except :
A Inert gas
B Minimal cardiovascular effects
C High blood solubility
D Rapid Induction and recovery
C

153.All are non-depolarizing muscle relaxants, except :
A Pancuronium
B Pipecuronium
C Cis-atracurium
D Succinyl choline
D
Suxamethonium is a depolarising muscle relaxant,

154. Conditions causing susceptibility to succinyl choline induced hyperkalemia,except :
A Burns
B Massive trauma
C Spinal cord injury
D Obesity
D
The administration of succinylcholine (SCh) in humans
results in a mild and transient hyperkalemia.
In normal individuals, the increase in serum
potassium (K1) is approximately 0.5–1.0 mEq/L, occurs
within 3–5 minutes after the IV administration of
SCh, and lasts ,10–15 minutes
This increase is
probably caused by K1 release from cells as a result of
depolarization at the neuromuscular junction. However,
in certain conditions, such as trauma, burns,
infection, and certain neuromuscular disorders (including
spinal cord injury, upper motor lesions, and
structural brain damage, peripheral nerve injury, peripheral
neuropathy, Parkinson’s disease, tetanus, and
muscular dystrophy), there is an exaggerated increase
in the serum K1 level that may manifest clinically in
cardiac dysrhythmias and even cardiac arrest . In
these conditions, it is believed that there is a proliferation
of postsynaptic acetylcholine receptors beyond
the neuromuscular junction (extrajunctional receptors)
with the result that K1 flux is not restricted to the
neuromuscular junction. This proliferation leads to an
exaggerated increase in serum K1 levels on depolarization
induced by SCh administration.

155. Regading post-dural puncture headache, all are true, except :
A Caused by decreased intracranial pressure
B The incidence is not related to needle size
C Epidural blood patch is effective as treatment
D Pregnant patients have higher incidence \&
B

156. False about propofol :
A Anti-convulsant
B Raises blood pressure
C Respiratory depression
D Decreases cerebral blood flow
B
Adverse effects of propofol
•Aside from the hypotension (mainly through vasodilatation) and transient apnea following induction doses, one of propofol's most frequent side effects is pain on injection, especially in smaller veins. This pain can be mitigated by pretreatment with lidocaine.Patients tend to show great variability in their response to propofol, at times showing profound sedation with small doses. A more serious but rare side effect is dystonia. Mild myoclonic movements are common, as with other intravenous hypnotic agents. Propofol appears to be safe for use in porphyria, and has not been known to trigger malignant hyperpyrexia.
•Another recently described rare, but serious, side effect is propofol infusion syndrome. This potentially lethal metabolic derangement has been reported in critically-ill patients after a prolonged infusion of high-dose propofol in combination with catecholamines and/or corticosteroids
propofol induces respiratory depression by inhib- iting the central inspiratory drive
Propofol has anticonvulsant activity which is likely mediated by yaminobutyric acid

157. Which is not true of doxapram ?
A.Useful in chronic obstructive pulmonary disease .
B.Has both central and peripheral action
C.It is a specific reversal agent
D.Should be avoided in epilepsy
Ans c

•Doxapram stimulates chemoreceptors in the carotid arteries, which in turn, stimulates the respiratory centre in the brain stem.
•Doxapram is used in intensive care settings to stimulate the respiratory rate in patients with respiratory failure. It may be useful for treating respiratory depression in patients who have taken excessive doses of drugs such as buprenorphine which may fail to respond adequately to treatment with naloxone.
•It is equally effective as pethidine in suppressing shivering after surgery
Side effects
•High blood pressure, panic attacks, tachycardia (rapid heart rate), tremor, sweating and vomiting may occur. Convulsions have been reported.
•It cannot be used in patients with coronary heart disease, epilepsy and high blood pressure.
•It is also contraindicated in newborns and small children, mainly due to the presence of benzyl alcohol.
•Doxapram is the generic name of the drug Dopram, used as an injection by IV or infusion, to restore respiration and breathing in a patient; often used in COPD patients. As a general rule, the use of doxapram occurs within a healthcare setting, under physician monitoring, with dosing taking as long as two hours. For patients who present to the emergency room with breathing complications associated with conditions such as COPD, doxapram provides a restoration to their ability to breath with a slow and meticulous process

158.An elevated level of serum creatine Kinase is found in :
A SLE
B DLE
C Dermatomyositis
D Scleroderma
C
•Muscle enzyme levels are often abnormal during the course of dermatomyositis (DM), except in patients with the amyopathic variant.
•The most sensitive/specific enzyme is elevated creatine kinase (CK), but aldolase studies and other tests (eg, aspartate aminotransferase [AST], lactic dehydrogenase [LDH]) may also yield abnormal results. At times, the elevation of the enzymes precedes clinical evidence of myositis. Thus, if a patient who is presumably stable develops an elevation of an enzyme previously within the reference range, the clinician should assess the possibility of a flare of the muscle disease.

159.Lepromin test is highly positive in :
A Tuberculoid Leprosy
B Borderline Leprosy
C Lepromatous Leprosy C
D Histoid Leprosy
A

160. Lisch Nodules have crucial diagnostic value in :
A Tuberous sclerosis
B Neurofibromatoses
C Xeroderma pigmentosum
D Incontinentia Pigmenti
B
Lisch nodules are the most common type of ocular involvement in neurofibromatosis-1.

161. The most effctive drug in the treatment of subcorneal pustular dermatosis is : A Azithromycin
B Dapsone
C Chloroquine
D Cyclosporine
•Subcorneal pustular dermatosis was first described by Sneddon and Wilkinson in 1956. It is a rare, benign, chronic relapsing sterile pustular eruption typically involving the flexural sites of the trunk and proximal extremities.
•It most commonly affects woman aged 40 years or older. The etiology of this entity is unknown, and its exact nosologic classification is still controversial. Reports suggest that some cases of subcorneal pustular dermatosis represent a variant of pustular psoriasis. In addition to this, recent studies have identified a possible subgroup with autoantibodies, known as subcorneal pustular dermatosis type immunoglobulin A (IgA) pemphigus. Some authors consider this subgroup to be a rare variant of pemphigus rather than subcorneal pustular dermatosis
•Dapsone is the treatment of choice. The response is slower than that seen with dermatitis herpetiformis, with resolution usually occurring in about 4 weeks. Once disease control has been established, the dose should be tapered to the lowest dose needed to maintain control. Sulfapyridine and sulfamethoxypyridazine may also be used, but only a few isolated reports support their effectiveness.
•Acitretin (and formally etretinate) has been used to successfully treat subcorneal pustular dermatosis and should be considered an alternative or additional treatment for those who are intolerant of or unresponsive to dapsone. Once disease control has been established, the dose should be tapered to the lowest dose needed to maintain control. Isotretinoin at 0.5 mg/kg/d appears to be ineffective.
•Phototherapy with psoralen with UVA, broadband UVB, and narrowband UVB alone or in combination with dapsone and/or retinoids can be successful at controlling subcorneal pustular dermatosis. Long-term maintenance regimens may be needed.
•Anecdotal case reports support the use of infliximab, tacalcitol, mizoribine, ketoconazole, tetracycline, minocycline, benzylpenicillin, vitamin E, azithromycin, cyclosporine, colchicine, and adalimumab with mycophenolate mofetil.
•Systemic and topical corticosteroids are generally ineffective but may provide some control. They have been used in combination with dapsone to treat associated conditions such as pyoderma gangrenosum and multiple myeloma. A good response to systemic corticosteroids is atypical and is suggestive of a diagnosis of pustular psoriasis.

162. Langerhan's_cells^ play a crucial role in :
A Healing of wound
B Distribution of Melanin pigment
C Contact sensitization
D Body temperature regulation
A
•Langerhans cells are dendritic cells[1] in the epidermis,containing large granules called Birbeck granules. They are normally present in lymph nodes and other organs, including the stratum spinosum layer of the epidermis. They can be found elsewhere, particularly in association with the condition histiocytosis.
•On infection of an area of skin, the local Langerhans cells will take up and process microbial antigens to become fully-functional antigen-presenting cells.
•Generally, dendritic cells in tissue are active in the capture, uptake and processing of antigens. Once dendritic cells arrive in secondary lymphoid tissue however, they lose these properties while gaining the capacity to interact with naive T-cells.
•Langerhans' cells are derived from the cellular differentiation of monocytes with the marker "Gr-1" (also known as "Ly-6c/G"). The differentiation requires stimulation by colony stimulating factor (CSF)-1.They are similar in morphology and function to macrophages.
•Langerin is a protein found in Langerhans cells,[5] and other types of dendritic cells
•The Langerhans cell is a specialized epidermal macrophage that may instigate wound healing via production of nitric oxide and epidermal growth factor

163. The incubation period of primary syphilis (chancre) is :
A.3 days – 6 days
B.9 days – 90 days
C.6 months – 7 months
D.8 months – 9 months
B
1.Primary syphilis: incubation period 2-3 weeks (range 9-90 days):

164. Molluscum contagiosum is caused by :
A.Staphylococci
B.Pox virus
C.The itch Mite
D.Human Papilloma virus
B

165. In Hepatocellular Carcinoma which one is incorrect ?
A. The history is important in evaluating predisposing factors
B. Alfa-fetoprotein is increased around half of the patients
C. Ultrasound examination of the liver is not a very definitive screening tool
D. Early stage tumours are successfully treated
D
Hepatic Ultrasound
•Limitations in the sensitivity and specificity of AFP in surveillance of high risk populations led to the use of ultrasound as an additional method for detection of hepatocellular carcinoma.[3] Studies in both healthy hepatitis B surface antigen carriers [4] and in patients with cirrhosis [6] have defined the performance characteristics of ultrasound as a screening test for HCC. Sensitivity in the former was 71% and in the latter 78%, with 93% specificity. The positive predictive values were 14% and 73%, respectively.
•Studies in patients with cirrhosis suggest that CT may be a more sensitive test for HCC than ultrasound or AFP > 20
Alfa-Fetoprotein (AFP)
•The reported sensitivity of AFP for detecting HCC varies widely in both HBV-positive and HBV-negative populations which is attributable to overlap between screening and diagnosis study designs.[3] When used for screening of high risk populations, a sensitivity of 39% to 97%, specificity of 76% to 95%, and a positive predictive value of 9% to 32% have been reported.[4-7] AFP is not specific for HCC. Titers also rise in acute or chronic hepatitis,[8] pregnancy, and in the presence of germ cell tumors.
About 70% of patients with hepatocellular carcinoma have elevated blood alpha-fetoprotei
Prognosis
•Overall prognosis for survival depends on the extent of cirrhosis and tumor stage, which then determine the appropriate treatment. Patients able to undergo a curative resection have a median survival of as long as 4 years; patients who present when they are too ill to be treated have a median survival of 3 months
ogeneral, the annual incidence of developing hepatocellular carcinoma in the setting of cirrhosis is approximately 1-4%. Screening studies have shown that, although lesions may be discovered at an earlier stage, the lack of curative treatment options in patients with cirrhosis may not lead to improvements in survival

166. Which one is incorrect among the following statements ?
A.Porophyrias are group of diseases having derngements of synthesis of haem
B.Porophyria Cutanea tarda is the most common type of human porphyria
C.Most patients with porphyria cutanea trada have associated mutations in the HFE gene linked to haemochromatosis
D. The Diagnosis is confirmed by demonstration of elevated levels of free ery-throcyte protoporphyrin
D
most common type of porphyria overall is porphyria cutanea tarda
•The HFE gene is located on the short (p) arm of chromosome 6 at position 21.3.
•HFE Mutations ARE SEEN IN
Hemochromatosis
•Porphyria
•X-linked sideroblastic anemia
Hemochromatosis
•Researchers have identified more than 20 mutations in the HFE gene that cause type 1 hemochromatosis. These mutations alter the size of the HFE protein or disrupt its 3-dimensional shape. As a result, the HFE protein cannot function properly. Two particular mutations are responsible for most cases of type 1 hemochromatosis. Each of these mutations changes one of the building blocks (amino acids) used to make the HFE protein. The most common mutation replaces the amino acid cysteine with the amino acid tyrosine at position 282 in the protein's chain of amino acids (written as C282Y or Cys282Tyr). The other mutation replaces the amino acid histidine with the amino acid aspartic acid at position 63 (written as H63D or His63Asp). As a result of these substitutions, the altered protein is not sent to the cell surface and does not interact with a cell surface receptor called the transferrin receptor. The transferrin receptor plays a critical role in regulating the amount of iron that enters the cell. When the HFE protein does not bind to the transferrin receptor, too much iron enters the body through the cells of the small intestine. This increased absorption of iron leads to the iron overload characteristic of this disorder.
porphyria - increased risk from variations of the HFE gene
•Mutations in the HFE gene that cause hemochromatosis are also believed to increase the risk of developing a form of porphyria called porphyria cutanea tarda. These mutations have been found more frequently in people with this condition than in unaffected people. Researchers are not certain how mutations in the HFE gene are related to the signs and symptoms of porphyria cutanea tarda. These mutations likely trigger this condition by increasing iron levels in the liver, as in hemochromatosis.
X-linked sideroblastic anemia - course of condition modified by mutations in the HFE gene
•One particular mutation in the HFE gene may increase the severity of symptoms in X-linked sideroblastic anemia when it is present in patients who also have mutations in the ALAS2 gene. The mutation results in a substitution of the amino acid tyrosine for the amino acid cysteine at position 282 (written as C282Y or Cys282Tyr) in the HFE protein. Although it is not known exactly how the mutation affects symptoms, it may cause iron to build up more rapidly in the body's tissues.
•Porphyria cutanea tarda (PCT) is a term encompassing a group of disorders in which activity of the heme synthetic enzyme uroporphyrinogen decarboxylase (UROD) is deficient
•Porphyria cutanea tarda includes familial types with UROD gene mutations and acquired types that may occur in individuals with a genetic predisposition (sporadic porphyria cutanea tarda), after exposure to hepatotoxins, or in the context of hepatic tumors. Familial porphyria cutanea tarda most often reflects the presence of one mutation at the UROD locus.
•Urinary porphyrin levels are abnormally high, with several hundred to several thousand micrograms excreted in a 24-hour period. The excess porphyrin pigment is often grossly evident in visible light and yields a pink fluorescence under Wood lamp (black light) radiation
• Chromatographic separation by carboxyl number of increased porphyrins present reveals a predominance of 8- and 7-carboxyl porphyrin fractions, with lesser amounts of 6-, 5-, and 4-carboxyl porphyrins, reflecting a UROD defect. A similar array of polycarboxylated porphyrins can be found in serum or plasma specimens. The fecal coproporphyrin fraction is often abnormally high and largely consists of isocoproporphyrin.
•Erythrocyte porphyrin levels are in the reference range, except in hepatoerythropoietic porphyria, in which zinc protoporphyrin is elevated

167. Among the followings which one is the incorrect statement ?
A. The human hemostatic system provides a natural balance between procoagulant and anti-coagulant forces
B. The platelet adhesion is primarily mediated by collagen tissue at the site of vascular injury
C. Platelet adhesion results in subsequent platelet adhesion and aggregation
D. The platelet GPIIBIIIA complex is the most abundant receptor on the platelet surface
C
The most abundant platelet aggregation receptor is glycoprotein (GP) IIb/IIIa
•All of the mutations that have been identified in patients with Glanzmann’s thrombasthenia result in a functional deficiency of GPIIb/IIIa receptors,2 3 and a hallmark of this disease is the absence of agonist-induced platelet aggregation.
•Collagen mediates platelet adhesion when subendothelial extracellular matrix proteins are exposed to blood,1and may play a role in atherothrombosis as well.
•Platelet adhesion to collagen is primarily mediated by 4 interactions: direct binding to a2ß1 and glycoprotein VI (GPVI) and indirect binding via collagen-bound von Willebrand Factor (vWF) bound to aIIbß3 and to glycoprotein Ib-IX-V

168. Which one is the incorrect statement among the following ?
A.Cutaneous lupus erythematous can be subdivided into acute, sub-acute and chronic types
B.Subacute lupus is characterised by erythema of the nose and malar eminences
C.Most of subacute cutaneous lupus erythematous have minimal or absence of renal CNS involvement
D.A Particular pattern of IgG deposits throughout the epidermis has been associated with subacute cutaneous lupus erythematus
B
•Subacute cutaneous lupus erythematosus (SCLE) is a nonscarring non–atrophy-producing photosensitive dermatosis.
•SCLE may occur in patients with systemic lupus erythematosus (SLE), Sjögren syndrome, and deficiency of the second component of complement (C2d), or it may be drug induced.
•Some patients also have the lesions of discoid lupus erythematosus (DLE), and some may develop small vessel vasculitis.
Papulosquamous lesions of subacute cutaneous lupus erythematosus may simulate psoriasis
•Subacute cutaneous lupus erythematosus (SCLE) often begins as a papular eruption.
–Papules may show a photosensitive distribution. Many patients notice that sun exposure results in an exacerbation of their disease, and some report worsening each spring and summer.
–Patients may complain of mild pruritus, but most patients are asymptomatic.
–Eventually, lesions develop into annular erythema or become psoriasiform in character.
•SCLE may wax and wane.
•Approximately 50% of SCLE patients have accompanying joint involvement.
–Arthralgias are common, often symmetrical, and usually affect small joints such as hands and wrists.
–Arthritis may occur but is unusual (<2%).>

169. Hypothermia is defined as when the body temperature goes below
A.< 96°F
B.< 95°F
C.< 94°F
D.< 90°F
B

1.Hypothermia is defined clinically as a core body temperature of less than 35 degrees Celsius (or 95 degrees Fahrenheit

170. In Gonococcal infection the following are true except :
A.Acute urethritis is the most common of clinical manifestation of gonorrhea in males
B.Gonococci become fully resistant to antibiotics either by Chromosomal mutations or by acquisition of R-Factors
C.Spectinomycin can be kept reserved for use against multiresistant strains of N. Gonorrhea
D.The vaginal mucosa in reproductive age is very much susceptible to Gonococcal infection
D
Lack of oestrogen, make vaginal mucosa more susceptible to irritation and infection with gonococci
Acute urethritis is the most common clinical manifestation of gonorrhea in males

171.In Ostogenesis Imperfecta (01) the following statements are correct, except :
A.Predominantly characterised by Osteopenia and brittle bones
B.OI can be classified into several varieties
C.The classifications mild and moderate forms are related to prognosis
D.The Sclerae can be normal in OI
C
•The following 4 types of OI have been reported:1
•Type I - Mild forms
•Type II - Extremely severe
•Type III - Severe
•Type IV - Undefined
in a small group of families the patients have normal sclerae and this disorder has been classified as Type 4 osteogenesis imperfecta
•Prognosis
•The life expectancy of subjects with nonlethal OI appears to be the same as that for the healthy population, except for those with severe OI with respiratory or neurologic complications.
•Patients with lethal OI may die in the perinatal period, but individuals with extremely severe OI can survive until adulthood.
•A classification system of different types of OI is commonly used to help describe how severely a person with OI is affected.

172. Regarding dementia the following statements are true, except :
A.More than 50% of the dementic patients suffer from Alzheimer's disease
B.Lumber puncture should be routinely done in the evaluation of dementia
C.Neuropsychiatric assessment is important for diagonsis
D.In early state of Alzheimer's disease mild depressive features, social withdrawal & are the most prominent psychiatric changes physical changes
B
Although uncommon in tests of dementia the lumbar puncture can reveal rare diseases that can mimic the signs of dementia.

173. In haematopoiesis the following statements are true, except :
A.Haematopoiesis is the process by which the formed elements of the blood are produced
B.In the bone Marrow the first morphologically recognizable erythroid precursor is the normoblast
C.The erythropoietin is produced by the peritubular cells within the kidney
D.There is daily replacment of 0.8 - 1% of all circulating red cells
B

174. The following statements are true except :
A.In acute care hospitals, 15-20% patients show evidence of significant malnutrition
B.Benefit can be offered to most of the patients by enteral or parenteral support
C.The timing of nutritional support is based on evaluation of the pre-existing nutritional status
D.The risks and benefits of specialised nutritional support are determined primarily by patient factors
A
Malnutrition. prevalence of 34% in acute care facilities
•The timing of nutritional support is based on evaluation of the preexisting nutritional status, the presence and extent of SRI,
Risks and Benefits of Specialized Nutrition Support. The risks are determined primarily by patient factors such as state of alertness, swallowing competence

175. In SLE which one is incorrect ?
A.Several environmental stimuli may influence SLE
B.SLE is a multigenic disease
C.Most of the auto-antibodies, characteristic of SLE of each person, are present at the time of clinical manifestations appear
D.The diagnosis is based on characteristic auto-antibodies
C
Systemic lupus erythematosus (SLE), a complex multigenic disease, is characterized by hypergammaglobulinemia, autoantibody production

176. In Diabetis Mellitus with dyslipedemia, the following statements are true, except :
A.Only very specific forms of dyslipidemia occurs
B.Almost all treatment studies of diabetic dyslipidemia are done with type-II D.M. C.Treatment is instituted to raise the HDL level
D.Combination therapy may be needed in Diabetic Dyslipedemia
C

177. Which one is incorrect regarding obsessive-compulsive disorder (GCD) ?
A.In OCD fears of contamination and germs are common
B.Obsessive-compulsion activities is taken up usually > 2 hours/day
C.Childhood onset is not rare
D.The anatomy of OCD is thought to include the orbital frontal cortex, caudate nucleus and globus pallidus
•person with OCD may spend 2-3 hours every day in the shower, and several more hours hand washing, or washing clothes, food and household items. Their anxiety may not only be that they are dirty themselves, but that they may infect others, contaminate foodstuffs and so forth. They may know that further washing is unnecessary, but they cannot stop the feeling of needing to wash and re-wash. Similarly, compulsions to check may involve repeatedly checking light and power switches to ensure that they are off, or checking locks to ensure that they are secure, despite knowing that they had just checked them.
•These compulsions and obsessions may take up many hours of a person's day. They can intrude into many routine activities and actions - for example, walking, eating, opening a door and reading may involve complex rituals.
•In actuality, childhood-onset OCD is fairly common, occurring in approximately 1% of all children

178. In HIV infection the following statements are true except :
A.Heart disease is a relatively common postmortem finding in HIV infected patients
B.Worldwide, approximately 1/3rd of all AIDS related deaths are associated with TB
C.The clinical manifestation of TB in HIV infected patients does not depend on CD4 + T cell counts
D.There are two forms of idiopathic interstitial pneumonia in HIV infected persons
C

179. Neutropenia is defined when the absolute neutrophil count goes below :
(A)< 2000/cu.m
(B)< 1500/cu.m
(C)< 1000/cu.m
(D)< 500/cu.m
D
1.Neutropenia is usually defined as when the neutrophil count goes below 500

180. The following statements are correct except one :
A.Hyperkalemia is defined as a plazma K + Concentration > 5 mml/l
B.Increase K+ intake is rarely the sole cause of hyperkalemia
C.Metabolic acidosis is always associated with mild hyperkalemia
D.Hyperkalemic periodic paralysis is a rare autosomal dominant disorder
C
•Hyperkalemic periodic paralysis (HYPP), also known as Impressive Syndrome, is an inherited autosomal dominant disorder which affects sodium channels in muscle cells and the ability to regulate potassium levels in the metabolic acidosis is often though not always associated with hyperkalemia
•Hyperkalemia, defined as a plasma K+ concentration >5.0 mmol/L, occurs as a result of either K+ release from cells or decreased renal loss.
•Increased K+ intake is rarely the sole cause of hyperkalemia since the phenomenon of potassium adaptation ensures rapid K+ excretion in response to increases in dietary consumption.
•Iatrogenic hyperkalemia may result from overzealous parenteral K+ replacement or in patients with renal insufficiency. Pseudohyperkalemia represents an artificially elevated plasma K+ concentration due to K+ movement out of cells immediately prior to or following venipuncture. Contributing factors include prolonged use of a tourniquet with or without repeated fist clenching, hemolysis, and marked leukocytosis or thrombocytosis. The latter two result in an elevated serum K+ concentration due to release of intracellular K+ following clot formation

181. Regarding jaundice, the following statements are true, except :
A Hall mark symptom of liver disease
B Reliable marker of severity
C Patients report scleral icterus before noticing darkening of urine
D Jaundice is rarely detectable with a bilirubin level 2.5 mg/dl
C
Generally, the elevation of transaminases is a reliable marker of liver disease
Level of serum AP is not a reliable indicator of the severity of underlying liver disease
•Jaundice: hallmark symptom of liver disease and perhaps the most reliable marker of severity
•Dark urine usually is reported before scleral icterus

182. The following statements are true, except :
A Polycythemia is defined as an increase in circulating red cells above normal
B History is urgent in the differential diagnosis
C Hematocrit level more than 50% irrespective of sex is abnomral
D Hematocrit more than 55% in women are abnormal
C
Polycythemia (erythrocytosis) is a condition characterized by an increase above normal in the number of red cells in the circulating blood

183. In pseudoaneurysm of the Aorta the following layers of the blood vessels are affected :
A The intima and media
B Intima,media and advertitia
C Media and adyentitia
D Intima and adventitia
C
A pseudoaneurysm, or false aneurysm, is an enlargement of only the outer layer of the blood vessel wall.
Pseudoaneurysms occur when the intima dilates into the media and adventitia

184. In thrombotic and Haemorrhagic strokes the following statements are true, except :
A.There is transient blood pressure elevation, lasting days to weeks
B.Specific blood pressure targets have been defined
C.Autoregulation of cerebral blood is impaired
D.Higher arterial pressures may be required to maintain cerebral blood flow
A
•Cerebral Perfusion Pressure (CPP) = MAP - ICP (Normal range 70 - 100 mmHg)
•Mean Arterial Pressure (MAP) = (SBP + 2 DBP)/3
•Intracranial Pressure (ICP) = 5- 15 mm Hg normally
For Intracerebral Hemorrhage patients:
•Rebleeding may occur if the Bp remains elevated
•Optimal target BP should take into account the premorbid degree of Hypertension
•Target Mean Arterial Pressure MAP of 120-130 mmHg
For Ischemic Stroke patients:
•Usually are hypertensive on admission
•Cerebral autoregulation is impaired
•Regional CBF (Cerebral Blood Flow) becomes pressure-dependent
•Elevated BP may be needed to aid blood flow to the ischemic prenumbra
•Target MAP of 120-140 mmHg
For Traumatic Brain Injury patients:
•Increased Bp may be secondary to a hyperadrenergic state vs elevated ICP
•Elevated ICP will decrease CBF
•Target MAP of 110-120 mmHg or a CPP of 70 mmHg
•Vital signs, while nonspecific, can point to impending clinical deterioration and may assist in narrowing the differential diagnosis. Many patients with stroke are hypertensive at baseline, and their blood pressure may become more elevated after stroke. While hypertension at presentation is common, blood pressure decreases spontaneously over time in most patients. Acutely lowering blood pressure has not proven to be beneficial in these stroke patients in the absence of signs and symptoms of associated malignant hypertension, AMI, congestive heart failure (CHF), or aortic dissection.

185. In Asthma among the following observations which one is incorrect ?
A.Atopy is the major risk factor for Asthma
B.The Hygiene hypothesis proposes that lower levels of infection in early childhood reduce the chance of Asthma
C.Duration of breast feeding may have direct effect with the occurrence of Asthma
D.Hyperventilation may trigger Asthma
B
•lack of exposure to infection in early life leads to development of atopic disease has come to be known as the hygiene hypothesis. It has arisen from observations of the rapidly rising prevalence of atopic diseases in recent decades and the lower prevalence of atopy with rising birth order
•the hygiene hypothesis.10 Expressed simply, this suggests that a reduced exposure to microbial pathogens in early life, especially those that result in a Th1 immune response, increases the chances of the expression of clinical atopic disease
exclusive breastfeeding may protect against asthma

186. The following are true in third degree heart block, except :
A.No atrial impulse propagates to the ventricles
B.A.V. Nodal Block is probably when the venticular rate is around 50/minute
C.The QRS complex is always wide
D.Pacemaker syndrome may develop if a simple ventricular pacemaker is used
C
•Pacemaker Syndrome is an iatrogenic disease that is often underdiagnosed.
–It represents the clinical consequences of suboptimal atrioventricular (AV) synchrony or AV dyssynchrony, regardless of the pacing mode, after the pacemaker plantation.[1]
–In general, the symptoms of the syndrome are a combination of decreased cardiac output, loss of atrial contribution to ventricular filling, loss of total peripheral resistance response, and nonphysiologic pressure waves
Symptoms commonly documented in patients history, classified according to etiology:[3][8]
•Neurological - Dizziness, near syncope, and confusion.[10]
•Heart failure - Dyspnea, orthopnea, paroxysmal nocturnal dyspnea, and edema.
•Hypotension - Seizure, mental status change, diaphoresis, and signs of orthostasis and shock.
•Low cardiac output - Fatigue, weakness, dyspnea on exertion, lethargy, and lightheadedness.
•Hemodynamic - Pulsation in the neck and abdomen, choking sensation, jaw pain, right upper quadrant (RUQ) pain, chest colds, headache.
•Arrhythmias - Palpitations.
•Heart rate related - Chest fullness or pain.
Risk Factors
•In the preimplantation period, two variables are predicted to predispose to the syndrome. First is low sinus rate, and second is a higher programmed lower rate limit. In postimplantation, an increased percentage of ventricular paced beats was the only variable that significantly predicted development of pacemaker syndrome by many studies
•Patients with intact VA conduction are at greater risk for developing pacemaker syndrome. Around 90% of patients with preserved AV conduction have intact VA conduction, and about 30-40% of patients with complete AV block have preserved VA conduction. Intact VA conduction may not be apparent at the time of pacemaker implantation or even may develop at any time after implantation.
•Patients with noncompliant ventricles and diastolic dysfunction are particularly sensitive to loss of atrial contribution to ventricular filling, where that highly increase the chance of developing the syndrome. This includes patients with cardiomyopathy (hypertensive, hypertrophic, restrictive) and elderly individuals
•Other factors correlated with development of pacemaker syndrome include decreased stroke volume, decreased cardiac output, and decreased left atrial total emptying fraction associated with ventricular pacing
Medical Care
•For some patients who are ventricularly paced, usually the addition of an atrial lead and optimizing the AV synchrony usually resolves symptoms
•In patients with other pacing modes, other than ventricular pacing, symptoms usually resolve after adjusting and reprogramming of pacemaker parameters, such as tuning the AV delay, changing the postventricular atrial refractory period, the sensing level, and pacing threshold voltage. The optimal values of these parameters for each individual differ. So, achieving the optimal values is by experimenting with successive reprogramming and measurement of relevant parameters, such as blood pressure, cardiac output, and total peripheral resistance, as well as observations of symptomatology
•In rare instances, using hysteresis to help maintain AV synchrony can help alleviate symptoms in ventricularly inhibited paced (VVI) patients providing they have intact sinus node function. Hysteresis effects a reduction in the amount of time spent in pacing mode, which can relieve symptoms, particularly when the pacing mode is generating AV dyssynchrony.[
•If symptoms persist after all these treatment modalities, replacing the pacemaker itself is sometimes beneficial and can alleviate symptoms

187. In the mechanism of heart failure which one is incorrect ?
A.The Frank-Starling mechanism operates through an increase in afterload
B.Compensatory hypertrophy of ventricle occurs in haemodynamic overload
C.There are changes in the size, mass and configuration of the ventricle in ventricular remodelling
D.There is redistribution of subnormal cardiac output away from the skin, skeletal muscle and kidneys
A
Sympathetic activation tends to redistribute systemic blood flow output away from the skin, gut, kidneys, and skeletal muscle to the heart and brain

188. In Primary Pulmonary Hypertension (PPH) the following are true, except:
A.Digoxin may increase cardiac output and lower circulating levels of Epinephrine
B.Diuretic therapy is useful in reducing right ventricular volume overload
C.Physical stress has got no effect in PPH
D.Anticoagulant therapy is advocated in all patients of PPH
C
•Chronic anticoagulation with warfarin (Coumadin) is recommended to prevent thrombosis and has been shown to prolong life in patients with primary pulmonary hypertension.
•Patients with this condition are prone to thromboembolism because of sluggish pulmonary blood flow, dilated right heart chambers, venous insufficiency and relative physical inactivity. Maintaining an International Normalized Ratio of 1.5 to 2.0 is recommended. Other anticoagulants are also being studied.
•In addition to arterial blood gas analysis, exercise and overnight oximetry should be used to determine if supplemental oxygen might benefit a patient with pulmonary hypertension.
Chest pain in PPH is precipitated by exertion or psychological stress

189. In sleep apnoea the following are true, except:
A.There is day time somnolence
B.Altered cardiopulmonary function is a common condition
C.Incidence of pulmonary hypertension is common and generally moderate in degree
D.Positive air way pressure delivered through the face mask during sleep is the treatment of choice
C
•Only about 1.1% of patients with COPD develop pulmonary hypertension with no other disease to explain the high pressure. Sleep apnea is usually associated with only very mild pulmonary hypertension, typically below the level of detection.
•On the other hand Pickwickian syndrome (obesity-hypoventilation syndrome) is very commonly associated with right heart failure due to pulmonary hypertension.

190. In acute renal failure which one is incorrect ?
A.Protienuria of 500 mg/day suggests injury to the glomerular ultrafiltration barrier
B.In prerenal ARF, the sediment is characteristically acellular and contains trans parent hyaline casts
C.A wide serum and osmolar gap are clues to the diagnosis of Ethylene Glycol Ingestion
D.Imaging of the urinary tract by ultrasonography is useful to exclude post-renal
A
Imaging of the urinary tract by ultrasonography is useful to exclude postrenal ARF
In prerenal ARF, the sediment is characteristically acellular and contains transparent hyaline casts ("bland," "benign," "inactive" urine sediment).

191. In Zollinger-Ellison (ZE) syndrome all are true, except :
A.Gastric acid hypersecretion characteristically causes peptic ulcer disease
B.The commonest presenting symptom is gastroaesophageal reflux disease
C.The syndrome may be associated with MEN-1
D.Most gastrinomas in this syndrome are present in the duodenum
B
•The most frequent locations of the primary tumor are the pancreas and duodenum, followed by the nearby lymph nodes.
•More than 90% of gastrinomas are located in the gastrinoma triangle, which is bound by the porta hepatis, the neck of the pancreas, and the third part of the duodenum.
•Ectopic locations include the ovaries, bile duct, heart, and kidneys.
CT scan of the abdomen with contrast enhancement, arterial-phase image. Note the brightly enhancing mass in the wall of the duodenum medially
CT scan of the abdomen with contrast enhancement in a patient with gastrinoma. Note the marked thickening of the proximal gastric folds
Octreotide scan of the abdomen Note the 2 small areas of increased activity in the right upper abdomen corresponding to the 2 small, adjacent masses between the duodenal wall and the head of the pancreas
Double-contrast study obtained with a barium meal shows prominent gastric rugae and thickened mucosal folds in the duodenum and jejunum
•A high index of suspicion is needed to make an early diagnosis of Zollinger-Ellison syndrome. Presenting symptoms include the following5, :
–Abdominal pain is the most common symptom and is present in 75% of patients. Typically, the pain is located in the upper abdomen. Abdominal pain is reported more frequently by men and patients with the sporadic form of Zollinger-Ellison syndrome.
–Diarrhea occurs in 73% of patients with Zollinger-Ellison syndrome. Diarrhea is the most common symptom in patients with the familial form and in female patients. Both diarrhea and abdominal pain are present in more than 50% of patients.
–Heartburn is the third most common symptom and may be mistaken for the symptoms of gastroesophageal reflux disease.
–Other symptoms include nausea, vomiting, GI tract bleeding, and weight loss. GI tract bleeding frequently occurs in the upper GI tract and is present in 25% of patients.
–In patients with the familial form of Zollinger-Ellison syndrome, a history of nephrolithiasis, hypercalcemia, and pituitary disorders must be sought. A family history of nephrolithiasis, hyperparathyroidism, and gastrinoma may be present
•These tumors are often small and, if in the duodenum, are usually less than 1 cm in diameter.
•They can be multiple in 20-40% of cases (in patients with multiple endocrine neoplasia (MEN) 1 syndrome), and two thirds are malignant
•The symptoms of Zollinger-Ellison syndrome are secondary to hypergastrinemia, which causes hypertrophy of the gastric mucosa and leads to an increase in the number of parietal cells (as many as 6 times the reference range value) and increased maximal acid output.
•On its own, gastrin also stimulates acid secretion, resulting in increased basal acid secretion.
•The increased acid production leads to mucosal ulceration of the GI tract, diarrhea, or malabsorption. Malabsorption in patients with Zollinger-Ellison syndrome is usually multifactorial and is caused by direct mucosal damage (caused by increased stomach acid), inactivation of the pancreatic enzymes, and the precipitation of bile salts.

192. In tetanus the following are true, except :
A.C. Tetani is an anaerobic, gram positive, rod shaped, terminal spore
B.Tetanus is entirely preventable by immunization
C.There is increased release of GABA and Glycine at the presynaptic terminal
D.It is characterised by increased muscle tone and rigidity
C
Tetanus is almost entirely preventable through immunization

193. In haemolytic anaemia the following are true, except :
A.Polychromatophilia
B.Increased reticulocyte count
C.Haptoglopin may be absent in blood
D.Lactate dehydrogenace will be diminished in blood
D
Lab fidings in Hemolytic anemia
CBC count
•The test documents anemia, leukocyte counts, and differential counts.
•Platelet counts help to exclude an underlying infection or hematological malignancy. The platelet count is within the reference range in most hemolytic anemias.
•Thrombocytopenia can occur in systemic lupus erythematosus (SLE), chronic lymphocytic leukemia (CLL), and microangiopathic hemolytic anemia (defective prosthetic cardiac valves, thrombotic thrombocytopenic purpura [TTP], hemolytic uremic syndrome [HUS], and disseminated intravascular coagulation [DIC]). Thrombocytopenia associated with a positive direct Coombs test result is known as Evans syndrome.
Peripheral smear and morphological examination
•Identifies polychromasia, indicating RBC immaturity reticulocytosis (
•Demonstrates spherocytes, suggesting congenital spherocytosis or autoimmune hemolytic anemia
•Can identify schistocytes (fragmented RBCs), suggesting thrombotic thrombocytopenic purpura, hemolytic uremic syndrome, or mechanical damage Can help diagnose a concomitant underlying hematological malignancy associated with hemolysis (ie, chronic lymphocytic leukemia)
Spherocytes. One arrow points to a spherocyte; the other, to a normal RBC with a central pallor.
Spherocytes. One arrow points to a spherocyte; the other, to a normal RBC with a central pallor.
RBC indices
•They are performed when a CBC count is requested.
•A low mean corpuscular volume (MCV) and mean corpuscular hemoglobin (MCH) are consistent with a microcytic hypochromic anemia, which may occur in chronic intravascular hemolysis, eg, paroxysmal nocturnal hemoglobinuria (PNH).
•A high MCV is consistent with a macrocytic anemia. Macrocytosis usually is due to megaloblastic anemias but can occur in liver disease. A high number of reticulocytes also may cause a high MCH.
•A high MCH and mean corpuscular hemoglobin concentration (MCHC) suggest spherocytosis.
•Increased red blood cell distribution width (RDW) study
•This is usually performed when a CBC count is requested.
•An increased RDW is a measure of anisocytosis, which is likely in hemolytic anemia.
Reticulocyte count
•An increased reticulocyte count is a criterion for hemolysis but is not specific for hemolysis.
•An increase may be caused by blood loss or a bone marrow response to iron, vitamin B-12, or folate deficiencies.
•The reticulocyte count may be normal or low in patients with bone marrow suppression despite ongoing severe hemolysis.
Lactic acid dehydrogenase
•Serum LDH is a criterion for hemolysis. LDH is not specific because it is ubiquitous and can be released from the neoplastic cells of the liver or other damaged organs.
•Although an increase in LDH isozyme 1 and 2 is more specific for RBC destruction, these enzymes are also increased in patients with myocardial infarction.
Serum haptoglobin
•A low serum haptoglobin is a criterion for moderate-to-severe hemolysis.
•A decrease is more likely in intravascular hemolysis than in extravascular hemolysis, but it is an acute phase reactant.
•The presence of concomitant infection, other reactive states, or chronic hemolysis may mask the diagnosis by raising haptoglobin levels.
Indirect bilirubin
•Unconjugated bilirubin is a criterion for hemolysis, but it is not specific because an elevated bilirubin also may indicate Gilbert disease.
•With hemolysis, the level of indirect bilirubin usually is less than 4 mg/dL.
•Higher levels of indirect bilirubin indicate compromised hepatic function or cholelithiasis and hemolysis.
•Changes in the LDH and serum haptoglobin levels are the most sensitive general tests because the indirect bilirubin is not always increased.

194. In I.T.P. which one is incorrect ?
A.The onset following recovery from a viral exanthem is common in children
B.The indolent form of I.T.P. is more common in children
C.Acute I.T.P. is caused by immune complexes containing viral antigens
D.The differential diagnosis of I.T.P. includes aplastic anaemia, acute leukaemias or metastatic tumour
B
The acute form of ITP is more prevalent in children, usually following exanthems

195. Rupture of Graffian Follicle is considered for all the following, except :
A.Fertility control
B.Emergency contraception
C.I.U.I.
D.Calculation of EDD
B
Ovulation refers to thr rupture of Graffian Follicle to release egg in the ovary

196. To confirm the diagnosis of pregnancy all are required, except :
A.TVS
B.Serum Assay of Progesterone
C.Serum Prolactin
D.Chorion villous biopsy
D
During pregnancy, prolactin levels increase by 10 to 20 times.

197. Heterotropic pregnancy may be found in the following condition, except :
A.Ectopic pregnancy
B.IVF
C.GIFT
D.Multiple pregnancy
D
•The risk of ectopic pregnancy and heterotopic pregnancy (ie, pregnancies occurring simultaneously in different body sites) dramatically increases when a patient has used assisted reproductive techniques to conceive, such as in vitro fertilization (IVF) or gamete intrafallopian transfer (GIFT). In a study of 3000 clinical pregnancies achieved through in vitro fertilization, the ectopic pregnancy rate was 4.5%, which is more than double the background incidence.
• Furthermore, studies have demonstrated that up to 1% of pregnancies achieved through IVF or GIFT can result in a heterotopic gestation, compared to an incidence of 1 in 30,000 pregnancies for spontaneous conceptions.

198. Common indications for Trial of Labour (T.O.L.) are all, except :
A.Repeat Caesarean Section
B.Breech presentation
C.MRFHS
D.Mid Pelvic dystocia
Ans d
•Contraindications :
–(1) Associated midpelvic and outlet contraction.
–(2) Presence of complicating factors like elderly primigravida, malpresentation, postmaturity, post Caesarean pregnancy, pre-eclampsia, medical disorders like heart disease, diabetes, tuberculosis etc.
–(3) Where facilities for caesarean section is not available round the clock.

199. Causes of Pre Term Labour (P.T.L.) are all, except :
A.Bacterial Vaginosis
B.Cervical incompetence
C.Increase of IL-6
D.Uterine Anomolies
C
High risk factors:for Preterm labour
(A) History : There is an increased incidence of preterm labour in cases such as : (1) Previous history ofinduced or spontaneous abortion or preterm delivery. (2) Asymptomatic bacteriuria or recurrent urinarytract infection. (3) Smoking habits. (4) Low socio-economic and nutritional status.
(B) Complications in present pregnancy : May be due to maternal, fetal or placental.
•. Maternal: (a) Pregnancy complications — such as pre-eclampsia, antepartum haemorrhage, premature rupture of the membranes, polyhydramnios. (b) Uterine anomalies such as, cervical incompetence, malformation of uterus, (c) Medical and surgical illness — Acute fever, acute pyelonephritis, diarrhoea, acute appendicitis, toxoplasmosis and abdominal operation. Chronic diseases such as hypertension, nephritis, diabetes, decompensated heart lesion, severe anaemia, Low body mass index (LBMI). (d) Genital tract infection — bacterial vaginosis, beta-haemolytic streptococcus, bacteroides, chlamydia, mycoplasma.
Fetal: Multiple pregnancy, congenital malformations, intrauterine death.
Placental: Infarction, thrombosis, placenta praevia or abruption.
(C) Iatrogenic : Elective induction with wrong estimation of gestational period.
Idiopathic : (Majority) — Premature effacement of the cervix with hyper-irritable uterus and earlyengagement of the head are often associated. In the absence of any complicating factors, it is presumedthat there is premature activation of the same systems involved in initiating labour at term.

200. GDM are associated with all, except :
A.Neonatal malformation
B.Macrosomia
C.Renal agenesis
D.Pleural Pregnancy
A
•The potential candidates for GDM are :
–(a) Positive family history of diabetes (parents or sibling). Family history should include uncles, aunts and grand parents
–(b) Having a previous birth of an overweight baby of 4 kg or more
–(c) Previous stillbirth with pancreatic islet hyperplasia revealed on autopsy
–(d) Unexplained perinatal loss
–(e) Presence of polyhydramnios or recurrent vaginal candidiasis in present pregnancy
–(f) Persistent glycosuria
–(g) Age over 30
–(h) Obesity
–(i) Ethnic group (East Asian, Pacific island ancestry).
•The literature on the relationship between renal agenesis and maternal diabetes is mixed with several reporting an association (Nielsen 2004, Parikh et al., 2002; Ramos-Arroyo et al., 1992) while others do not (Moore et al., 2000; Becerra et al., 1990). One study found that high body mass index among women with gestational diabetes was associated with the occurrence of major renal/urinary defects (Garcia-Patterson 2004) , while another found higher risk for renal defects (several types) and a women who had higher body mass index along with a history of fertility problems (Honein 2003).
•HAZARDS:
–(1) Increased perinatal loss is associated with fasting hyperglycemia. Fetal anomalies are however not increased. This is due to the absence of metabolic disturbance during organogenesis.
–(2) Increased incidence of macrosomia.
–(3) Polyhydramnios.
–(4) Birth trauma.
–(5) Recurrence of GDM in subsequent pregnancies is about 50 percent.

201. The causes of obstetric cholestasis are none, except :
A.Deficit in MDR-3
B.Acute Cholecystitis
C.Hepatitis-B
D.Increase in Glutathione-S-Transferase
A
•INTRA HEPATIC CHOLESTASIS (Syn : Obstetric hepatosis, Icterus gravidarum) , •It is the second most common cause of jaundice in pregnancy, the first one being viral hepatitis. The
•stasis of bile in the bile canaliculi with rise in conjugated bilirubin is probably due to excess circulating oestrogen. Similar manifestation is also observed in women taking contraceptive pills.
•The manifestations usually appear in the last trimester. The onset is insidious; generalised pruritus is the predominant symptom; there may be weakness, nausea or even vomiting. Jaundice is slight. There is rise in the levels of AST, ALT and serum alkaline phosphatase. Bilirubin level rarely exceeds 5 mg%. Liver biopsy shows no evidence of necrosis but shows the features of intrahepatic cholestasis.
•Progressive familial intrahepatic cholestasis (PFIC) is a chronic cholestasis syndrome that begins in infancy and usually progresses to cirrhosis within the first decade of life.
•The average age at onset is 3 months, although some patients do not develop apparent cholestasis until later, even as late as adolescence.
•PFIC can progress rapidly and cause cirrhosis during infancy, or it may progress relatively slowly with minimal scarring well into adolescence. Few patients have survived into the third decade of life without treatment.
•The condition clinically characterized by hepatocellular cholestasis, low serum levels of gamma-glutamyl transferase (GGT) activity, and autosomal recessive inheritance is termed low-GGT PFIC
MDR3 is involved in canalicular excretion of phospholipids
•MDR-3 is a primary active export pump that belongs to the family of ABC transporters and is expressed in the canalicular membrane of the hepatocyte.
•It functions in the translocation of phosphatidylcholine across the canalicular membrane.
•Mdr-2 knockout mice and MDR-3(-) humans cannot excrete this phospholipid into bile.
•Both develop progressive liver disease characterized by portal inflammation, proliferation of bile ducts, and fibrosis. Mdr-2–deficient mice made transgenic by expression of the human homologue of Mdr-2 (ie, MDR-3) recover function and excrete phospholipid in their bile. This finding confirms the functional homology between the mouse and human genes and further suggests that phospholipid excretion is limited by the amount of MDR-3 or Mdr-2 present.
•There is increased incidence of preterm labour, low birth weight babies, meconium stained liquor, IUD and postpartum haemorrhage. The features subside within two weeks postpartum. Treatment consists of diphenhydramine (antihistamine) for intense pruritus. Cholestyramine is effective for itching. In that case, patient should also have vit K, 10 rhg/day. Prothombin time should be monitored. Ursodeoxycholic acid (UDCA) is found helpful. Recurrence rate is high (50-60%).

202. Pregnancy associated with Breast Cancer is noted in the the following conditions, except:
A.BRCA1
B.Mutated P-53 gene
C.Galactocele
D.Lactating Adenoma
C, d
Lactating adenomas are the most prevalent breast masses seen in pregnant
Lactating adenoma is a benign breast lesion \ Lactating adenomas are not thought to carry an increased risk of breast carcinoma

203. Factors controlling the outcome of IVF depends on all, except :
A.Types of Gronadotropin used
B.Number of Embryo Transfer
C.Patients' monitoring
D.Embryo transfer technique
C
The main factors affecting IVF outcome include the following:
•age of the woman ( and consequently, her ovarian reserve )
•normalcy of the uterus, and semen quality
•success or failure of fertilization and cleavage in vitro
•number of embryos transferred and cryopreserved
•adequacy of the luteal phase after transfer

204. The prognosis of endometrial carcinoma depends on all, except :
A.DNA Ploidy
B.Cervical glandular involvement
C.Oncogene mutation
D.Receptor status
B

205. Caesarean myomectomy is a more feasible undertaking if the following condition are fulfilled, except :
A.Full consent of the patient
B.Experienced surgeon
C.Fibroid in the upper segment
D.Lower segment not approachable
C
Myomectomy during caesarean section is conventionally reserved for pedunculated myomas
•in the setting of the symptomatic patient with an accessible subserosal or pedunculated fibroid or the patient with fibroids obstructing the lower uterine segment this procedure can be safely accomplished

206. Investgations for ureteric injury are all, except :
A.Intravenous Pyelography
B.Retrograde Ureterogram
C.Fistulogram
D.USG of Pelvis
D
Retrograde ureterogram
When the results of an IVU and CT scan are
inconclusive, a retrograde ureterogram may be
necessary to evaluate the course of the ureter.
This identifies the anatomic site of obstruction,
even when missed on an IVU or CT scan, by
delivering a higher density of contrast material
to the injured site. A retrograde ureterogram is,
however, more invasive than either an IVU or
CT scan and requires cystoscopy.
Renal ultrasound
Renal ultrasound is perhaps the best noninvasive method to visualise the kidney. Hydronephrosis and retroperitoneal urinomas are shown with great sensitivity. Renal ultrasound cannot, however, assess either renal function or continuity of the ureter.
Contrast-dye tests
Contrast-dye tests are normally combined with
cystoscopy and may be particularly useful if a urinary
fistula is suspected.The path of the fistula can
usually be determined by the simultaneous administration
of intravenous indigo carmine (a blue dye) and placement in the bladder of Congo red
through a transurethral catheter.

207. Effective use of LNG-IUCD is only in :
A.Prevention of PID
B.Prevention of ovarian cyst
C.Prevention of Ectopic Pregnancy
D.Contraception
D
The LNg IUD can be inserted early in a woman's reproductive life and its contraceptive effect is reversible.

208. Surgery in the treatment of Ovarian Malignancy is the corner stone, except :
A.Diagnosis
B.Staging
C.Prognosis
D.Screening
D

209. Useful molecular marker in Cervical Cancer Diagnosis are all, except :
A.CEA
B.Vimentin (VIM)
C.Chromogranin A
D.Synaptophysin
D
CEA is elevated in about 20 percent of all women with cervical carcinomas
Vimentin expression has been demonstrated to correlate with a more aggressive phenotype in human cervical cancer
•Chromogranin A might promote the generation of secretory granules. It is used as an indicator for pancreas and prostate cancer and in carcinoid syndrome
•The protein is a synaptic vesicle glycoprotein with four transmembrane domains weighing 38kDa. It is present in neuroendocrine cells and in virtually all neurons in the brain and spinal cord that participate in synaptic transmission. It acts as a marker for neuroendocrine tumors, and its ubiquity at the synapse has lead to the use of synaptophysin immunostaining for quantification of synapses

210. Development of Placenta is from none, except :
A.Decidua Basalis
B.Chorionic Villi
C.Rohr's Stria
D.Chorion Frondosum
A
•That part of the decidua that interacts with the trophoblast is the decidua basalis.
•Decidua is the term for the uterine lining (endometrium) during a pregnancy, which forms the maternal part of the placenta. It is formed under the influence of progesterone and forms highly-characteristic cells.

241. In which of the following cyanotic heart disease shows enlarged cardiac shadow and plethoric lung field in chest X-ray :
A VSD
B TOF
C TGA
D Ebstein's anomaly.
C
•Transposition of the great arteries (TGA) is the most common cyanotic congenital heart lesion that presents in neonates. The hallmark of TGA is ventriculoarterial discordance, in which the aorta arises from the morphologic right ventricle and the pulmonary artery arises from the morphologic left ventricle
The clinical course and manifestations depend on the extent of intercirculatory mixing and the presence of associated anatomic lesions.
TGA with intact ventricular septum:
Prominent and progressive cyanosis within the first 24 hours of life is the usual finding in infants. TGA with large ventricular septal defect
•Infants may not initially manifest symptoms of heart disease, although mild cyanosis (particularly when crying) is often noted.
•Signs of congestive heart failure (tachypnea, tachycardia, diaphoresis, and failure to gain weight) may become evident over the first 3-6 weeks as pulmonary blood flow increases.
TGA with ventricular septal defect and left ventricular outflow tract obstruction
•Infants often present with extreme cyanosis at birth, proportional to the degree of left ventricular (pulmonary) outflow tract obstruction.
•The clinical history may be similar to that of an infant with tetralogy of Fallot.
TGA with ventricular septal defect and pulmonary vascular obstructive disease
•Progressively advancing pulmonary vascular obstructive disease can prevent this rare subgroup of patients from developing symptoms of congestive heart failure, despite a large ventricular septal defect.
•Most often, patients present with progressive cyanosis, despite an early successful palliative procedure.
Chest radiography
•The chest radiograph may appear normal in newborns with transposition of the great arteries (TGA) and intact ventricular septum but may demonstrate the classic "egg on a string" appearance in approximately one third of patients.
•With a ventricular septal defect, cardiomegaly usually occurs with increased pulmonary arterial vascular markings.
The heart is enlarged with a narrow "pedicle" giving the so called "egg on a string" appearance. The superior mediastinum appears narrow due to the antero-posterior relationship of the transposed great vessels and "radiologic-absence of the thymus".

242. In Duchenne Muscular dystrophy,the following muscles appear large,except : A Sternal head of the pectoralis major
B Glutei
C Deltoid
D Tongue muscle
A
In DMD -Pseudohypertrophy can be seen in TONGUE gluteal, deltoid and masseter muscles as well as the calves

243. Reye's syndrome is a :
A Primary mitochondrial Hepatopathy
B Secondary mitochondrial hepatopathis
C Bile acid biosynthesis abnormalities
D Type 2 autoimmune hepatitis
B

244. Congenital anomalies associated with Wilm's tumour all of the following, except :
A Beck Weith-Wiedmann syndrome
B Hemihypertrophy
C Aniridia
D None of the above
D
•Patients with an unusual complex of congenital developmental abnormalities, such as aniridia, genitourinary (GU) malformations, and mental retardation, are at high risk (>30%) of having a Wilms tumor. At birth, the association is aniridia, GU malformations, and mental retardation (AGR) syndrome. With the discovery of a Wilms tumor in these patients, the association is referred to as WAGR syndrome. These syndromes result from the loss of chromosomal material from the short arm of chromosome 11.
•Denys-Drash syndrome (DDS) is a rare disorder consisting of the triad of congenital nephropathy, Wilms tumor, and intersex disorders resulting from mutations in the Wilms tumor suppressor (WT1) gene. Nephropathy is a constant feature; in the incomplete forms of the syndrome, the nephropathy coexists with either Wilms tumor or intersex disorders, but the vast majority of patients with DDS are destined to develop Wilms tumor in any residual renal tissue. The characteristic nephropathy in DDS is termed diffuse mesangial sclerosis. This condition manifests clinically as an early onset nephrotic syndrome and progresses to renal failure during the first 3 years of life. Among the intersex disorders, pure gonadal dysgenesis with male pseudohermaphroditism is the classic presentation, although a wide variety of abnormalities in gonadal differentiation can be encountered
•congenital exomphalos, macroglossia, and gigantism.
•BWS is a congenital disorder.
–Wilms tumor is the most common cancer in children with BWS, occurring in about 5-7% of all children with BWS. Most children develop Wilms tumor before age 4 years; however, children with BWS can develop Wilms tumor when they are as old as 7 or 8 years. By age 8 years, 95% of all Wilms tumor cases have been diagnosed

245. Least common cause of persistent hypertension in children :
A Chronic glomerulonephritis
B Chronic pyelonephritis
C Pheochromocytoma
D Essential hypertension
D
•Essential Hypertension (EHT) is rare in children.
•The reported prevalence of mild EHT in the paediatric age group is 1-5%

246. All of the following condition causes microcephaly, except :
A Edward syndrome
B Alexander disease
C Down syndrome
D Rubella syndrome
B
•Alexander disease is a slowly progressing and fatal neurodegenerative disease. It is a very rare disorder which results from a genetic mutation and mostly affects infants and children, causing developmental delay and changes in physical characteristics
•Delays in development of some physical, psychological and behavioral skills, progressive enlargement of the head (macrocephaly), seizures, spasticity, in some cases also hydrocephalus, dementia.
•Alexander disease is a genetic disorder affecting the central nervous system (midbrain and cerebellum).
•It is caused by mutations in the gene for glial fibrillary acidic protein (GFAP) that maps to chromosome 17q21. It is inherited in an autosomal dominant manner.
•Alexander disease belongs to leukodystrophies, a group of diseases which affect growth or development of the myelin sheath. The destruction of white matter in the brain is accompanied by the formation of fibrous, eosinophilic deposits known as Rosenthal fibers.
Case presentation
•MR imaging of a patient with biopsy-confirmed infantile Alexander disease
•At the age of 1 months, the frontal white matter has a slightly higher signal intensity on T2-weighted images and slightly lower signal intensity on unenhanced T1-weighted images than does the remainder of the cerebral white matter, which has normal signal intensity for unmyelinated white matter. There is a periventricular rim of low signal intensity on T2-weighted images (arrows, A) and high signal intensity on T1-weighted images (arrows, B), with some extensions into the frontal white matter (arrowheads, A and B). The caudate nucleus and putamen have high signal intensity on T2-weighted images and are mildly swollen.

247. Lissencephaly means :
A Presence of bilateral clefts within the Cerebral hemisphere
B Developmental defective cleavage of the prosencephalon
C Absence of cerebral convolutions and a poorly formed sylvian fissurae
D Agensis of the corpus collosum
C
•Lissencephaly, which literally means smooth brain, is a rare brain formation disorder caused by defective neuronal migration during the 12th to 24th weeks of gestation, resulting in a lack of development of brain folds (gyri) and grooves (sulci).[1] It is a form of cephalic disorder.
•Terms such as 'agyria' (no gyri) or 'pachygyria' (broad gyri) are used to describe the appearance of the surface of the brain. Children with lissencephaly are severely neurologically impaired[2] and often die within several months of birth
•Causes of lissencephaly can include viral infections of the uterus or the fetus during the first trimester, or insufficient blood supply to the fetal brain early in pregnancy. There are also a number of genetic causes of lissencephaly, including mutation of the reelin gene (on chromosome 7), as well as other genes on the X chromosome and on chromosome 17
Axial and sagittal PD/T2WI show uniformly thickened cortex with only minimal sulcation of the frontal lobes. The sylvian fissures are widened. Parietooccipital sulcation is absent.

248. In pyogenic meningitis sensorineural hearing loss is most commonly seen in which type:
A Pneumococcal meningitis
B Meningococcal meningitis
C H influenzae type B meningitis
D Staphylococcal meningitis
A
•Before the introduction of the conjugate vaccine against H influenza type b, H influenza type b was one of the leading causes of bacterial meningitis.
•Currently, S pneumoniae and N meningitidis are the most common organisms responsible for bacterial meningitis

249. Which of the following is a correct ingradients in WHO ORS formula (mmol/L) ?
A Sodium 80
B PotassiunT30
C Chloride 80
D Citrate 30
A
Switch to reduced osmolarity ORS
•In 2003, WHO/UNICEF changed the ORS formula to a reduced osmolarity version from what it had been recommending for over two decades prior.This change was in response to numerous studies that showed that the standard ORS formula was ineffective in reducing diarrheal stool output compared to other solutions, including rice water.
•Additionally, further studies showed that a reduced osmolarity solution not only decreased stool output, but also resulted in less vomiting and fewer unscheduled intraveneous therapy.
•Although UNICEF certifies reduced osmolarity ORS for all forms of dehydration,at least one study cautions that for high stool output cholera-based diarrhea, reduced osmolarity ORS may not sufficiently replenish electrolyte levels, leading to hyponatremia. Though the actual consequence of this appeared negligible, further study was recommended.
•The change reduced the osmolarity of the ORS solution from 311 mmol/L to 245 mmol/L.
•The ingredients reduced in concentration were glucose and sodium chloride. Potassium and citrate concentrations remained the same.[
•The benefits of the reduced osmolarity ORS are reducing stool volume by about 25%, reducing vomiting by nearly 30%,[25] and reducing the need for unscheduled intraveneous therapy by 33%

250. Wide and variable splitting of the 2nd heart sound seen in all of the following, except :
A MR
B VSD without pulmonary pypertension
C Tetralogy of Fallots
D PS
B
1. delayed electrical activation of the right ventricle
–(as in complete right bundle branch block and premature ventricular beats, and with left ventricular pacemakers)
–2. prolonged right ventricular or shortened left ventricular ejection time (as in valvular or infundibular pulmonic stenosis, mitral regurgitation, and ventricular septal defects)
–3. altered impedance of the pulmonary vascular bed (massive pulmonary embolism).
Wide and fixed splitting of S2 (splitting of S2 that does not vary with respiration) is found in patients with large atrial septal defects, severe pulmonary stenosis, and right ventricular failure.Normally splitting is more during inspiration when blood flow to the right heart increases due to negative intrathoracic pressure. In ASD this increase is compensated by left to right to shunt during expiration. So during whole cardiac cycle a fixed amount flows through pulmonic valve making the variation in split very small.
•Paradoxical splitting of S2 (when splitting increases with expiration) is usually due to a delayed A2. This delay in A2 may be due to1. electrical conduction disorders (complete left bundle branch block, right ventricle premature contractions, and ventricular tachycardia) or2. mechanical disorders (severe valvular aortic stenosis, left ventricular outflow obstruction, hypertrophic cardiomyopathy, coronary artery disease, myocarditis, and congestive cardiomyopathy).
Infants with small defects - -VSD
•Patients may have normal vital signs.
•Physiologic splitting of S2 is usually retained.
•The characteristic harsh, holosystolic murmur is loudest along the lower left sternal border (LSB), and it is well localized. Small defects can produce a high-pitched or squeaky noise.
•The murmur is usually detected after the PVR decreases at about 4-8 weeks of age.
Infants with moderate defects -VSD
•Infants often have a normal length and decreased weight. Poor weight gain is a sensitive indicator of congestive heart failure (CHF).
•Infants may have mild tachypnea, tachycardia, and an enlarged liver.
•The precordial activity is accentuated.
•The murmur with moderate-sized defects is usually associated with thrill. A holosystolic harsh murmur is most prominent over the lower LSB.
•The intensity of the pulmonary component is usually normal or slightly increased.
•In addition to the harsh holosystolic murmur, a diastolic rumble may be detected in the mitral area. This rumble suggests functional mitral stenosis secondary to a large left-to-right shunt and indicates a surgical-level shunt (pulmonary-to-systemic flow ratio [Qp:Qs], 2:1)
Infants with large ventricular septal defects - -VSD
•As with moderate defects, signs of CHF are present. The cardinal signs of heart failure include tachycardia, tachypnea, and hepatomegaly. In addition, cardiomegaly is present and helps in differentially diagnosing heart failure as opposed to a respiratory condition, such bronchiolitis.
•The murmur is holosystolic but poorly localized and is usually associated with a diastolic rumble. A ventricular septal defect is not typically associated with cyanosis. It is a "pink" condition; persistent cyanosis from birth indicates a relatively complicated lesion than isolated ventricular septal defect. The occurrence of cyanosis after infancy suggests reversal of the shunt. Patients with large ventricular septal defects and marked elevations of PVR frequently appear well in childhood because the blood flow in their systemic and pulmonary circuits is well balanced.

251. Which of the following is not commonly seen in a patients of chronic renal failure ?
A Growth failure
B Normocytic,normochromic anemia
C Hypertension
D Secondary hypoparathyroidism
D
•Chronic renal failure is the most common cause of secondary hyperparathyroidism. Failing kidneys do not convert enough vitamin D to its active form, and they do not adequately excrete phosphorus. When this happens, insoluble calcium phosphate forms in the body and removes calcium from the circulation. Both processes leads to hypocalcemia and hence secondary hyperparathyroidism. Secondary hyperparathyroidism can also result from malabsorption (chronic pancreatitis, small bowel disease) in that the fat soluble vitamin D can not get reabsorbed. This leads to hypocalcemia and a subsequent increase in parathyroid hormone secretion in an attempt to increase the serum calcium levels.

252. The commonest organism for actue epiglottitis in children is :
A Staphylococcus
B H influenzae type B
C Pneumococcal
D RSV
B

253. Which of the following causes myocarditis, except :
A Diphtheria
B Influenzae virus
C Both of the above
D None of the above
C
influenza B infection IS associated myocarditis and severe skeletal myositis

254. Attention deficit hyperactivity disorder has all the following features, except :
A Hyperactivity
B Delusion
C Impulsivity
D Inattention
B
•The most common symptoms[28][29] of ADHD are:
–Impulsiveness: acting before thinking of consequences, jumping from one activity to another, disorganization, tendency to interrupt other peoples' conversations.
–Hyperactivity: restlessness, often characterized by an inability to sit still, fidgeting, squirminess, climbing on things, restless sleep.[28]
–Inattention: easily distracted, day-dreaming, not finishing work, difficulty listening

255. Feature suggestive of Bipolar depression are all,except :
A Late age of onset
B Psychotic depression
C Atypical features
D Seasonality
A ,D
•atypical features are an important marker of bipolar II disorder.
Clinical experience also suggests that psychotic depression in a young person is a common initial presentation of a bipolar illness
Criteria for a major depressive episode DSM-IV-TR
•Five (or more) of the following symptoms have been present during the same two-week period and represent a change from previous functioning; at least one of the symptoms is either (1) or (2).
•depressed mood most of the day, nearly every day, as indicated by either subjective report (e.g., feels sad or empty) or observation made by others (e.g., appears tearful). Note: In children and adolescents, can be irritable mood.
•markedly diminished interest or pleasure in all, or almost all, activities most of the day, nearly every day (as indicated by either subjective account or observation made by others)
•significant weight loss when not dieting or weight gain (e.g., a change of more than 5% of body weight in a month), or decrease or increase in appetite nearly every day. Note: In children, consider failure to make expected weight gains.
•Insomnia or Hypersomnia nearly every day
•psychomotor agitation or retardation nearly every day (observable by others, not merely subjective feelings of restlessness or being slowed down)
•fatigue or loss of energy nearly every day
•feelings of worthlessness or excessive or inappropriate guilt (which may be delusional) nearly every day (not merely self-reproach or guilt about being sick)
•diminished ability to think or concentrate, or indecisiveness, nearly every day (either by subjective account or as observed by others)
•recurrent thoughts of death (not just fear of dying), recurrent suicidal ideation without a specific plan, or a suicide attempt or a specific plan for committing
•DSM-IV-TR diagnostic categories and criteria for Bipolar Disorder
–Bipolar I
–Bipolar II
–Cyclothymia
–Bipolar Disorder NOS (Not Otherwise Specified)

256. Defence mechanisms described in Obsessive-Compulsive Disorder are all, except :
A Reaction formation
B Undoing
C Isolation
D Conversion
D
•Defense mechanisms in OCD of isolation, undoing, reaction formation, ambivalence, magical thinking.

257. All are Somatoform disorders, except :
A Somatization disorder
B Conversion disorder
C Amnestic disorder
D Body dysmorphic disorder
C
•Somatoform disorder (also known as Briquet's syndrome) is characterized by physical symptoms that mimic disease or injury for which there is no identifiable physical cause or physical symptoms such as pain, nausea, depression, and dizziness.
The Somatoform disorders recognized by the Diagnostic and Statistical Manual of Mental Disorders of the American Psychiatric Association are:
•Conversion disorder
•Somatization disorder
•Hypochondriasis
•Body dysmorphic disorder
•Pain disorder
•Undifferentiated somatoform disorder - only one unexplained symptom is required for at least 6 months.
•Included among these disorders are false pregnancy, psychogenic urinary retention, and mass psychogenic illness (so-called mass hysteria).
•Somatoform disorder NOS

258. All are Cluster B personality disorders, except :
A Emotionally unstable personality disorder
B Anankastic personality disorder
C Histrionic personality disorder
D Antisocial personality disorder
B
•The DSM-IV lists ten personality disorders, grouped into three clusters. The DSM also contains a category for behavioral patterns that do not match these ten disorders, but nevertheless exhibit characteristics of a personality disorder. This category is labeled Personality Disorder NOS (Not Otherwise Specified).
Cluster A (odd or eccentric disorders)
•Paranoid personality disorder: characterized by irrational suspicions and mistrust of others
•Schizoid personality disorder: lack of interest in social relationships, seeing no point in sharing time with others
•Schizotypal personality disorder: also avoids social relationships, though out of a fear of people
Cluster B (dramatic, emotional, or erratic disorders)
•Antisocial personality disorder: "pervasive disregard for the law and the rights of others."
•Borderline personality disorder: extreme "black and white" thinking, instability in relationships, self-image, identity and behavior
•Histrionic personality disorder: "pervasive attention-seeking behavior including inappropriate sexual seductiveness and shallow or exaggerated emotions"
•Narcissistic personality disorder: "a pervasive pattern of grandiosity, need for admiration, and a lack of empathy"
Cluster C (anxious or fearful disorders)
•Avoidant personality disorder: social inhibition, feelings of inadequacy, extreme sensitivity to negative evaluation and avoidance of social interaction
•Dependent personality disorder: pervasive psychological dependence on other people.
•Obsessive-compulsive personality disorder (not the same as obsessive-compulsive disorder): characterized by rigid conformity to rules, moral codes, and excessive orderliness
•anankastic personality disorder, or obsessive-compulsive personality disorder is Class C personality disorder

259. Hallucination without any impairment of consciousness, and without any other psy-chopathology is called :
A Reflex hallucination
B Alcoholic hallucinosis
C Pseudo-hallucination
D Delirium tremens
C
•Reflex hallucination The experience of a real stimulus in one sensory modality triggering a hallucination in another
•Hallucination An internal perception without a corresponding external object. The subjective experience of hallucination is that of experiencing a normal perception in that modality of sensation. A true hallucination will be perceived as in external space, distinct from imagined images, outside conscious control, and as possessing relative permanence.
–A pseudo-hallucination will lack one or all of these characteristics.

260. Full name of CBT is :
A Cognitive Behavioural Therapy
B Counselling and Behavioural Treatment
C Career Building Technique
D Concurrent Behavioural Therapy
A
Cognitive Behavioural Therapy (CBT)



MD Entrance TEST-PREP
TARGET BATCH 2009-10
DLI – Hyderabad
Jammu Address :17 A/C Gandhi Nagar
Ph No 2431418, 9419124645
Srinagar Address : Lane opp A Bee Book Store, Chanapora
Ph no. 9419014881

Audio Video lectures series loadable on PC
1000 Hrs of lectures covering all 20 subjects
Discussion of 5000 Question databank of previous 10 years of MD questions of AIIMS, PGI etc.
Live lecture series from eminent faculty of city

All INDIA Test Series
30 All India Subject tests ( starting MAY – 2009) with All India ranking on fixed schedule to check your performance vis a vis 10000 doctors.
30 All India Full Length Tests on same lines.

DLI Hyderabad MD study Plan
For Program & Fee Details contact Mr. Vivek Sharma at 9419184183.
DLI claimed 1st rank at JK MD-08 as well as 67% success rate among JK MD enrolments with DLI - Jammu